теория информации, идеальный шифр и черные дыры


Один из основных вопросов физики и теории информации: исчезает ли в черных дырах информация? Известно, что они излучают равновероятный шум. Вопрос: если наложить на открытый текст равновероятную гамму, то ведь информация в сообщении не потеряется? Можно ли предположить, что в черных дырах идет подобный процесс: на информацию исходящую изнутри горизонта событийнакладывается равновероятный шум? Значит, инфлрмация не уничтожается?

Комментарии
Гость (16/08/2013 02:10)   

Если гамму после этого уничтожить, то потеряется.
Гость (16/08/2013 02:28)   
Нет, не исчезает.
Гость (21/08/2013 21:05)   
При измерении кубита, его первоначальное состояние бесследно исчезает. В квантовой механнике любое измерение это одновременно изменение состояния карпускулы.

Что происходит с карпускулами за горизонтом событий чёрной дыры, Шрёдингерскому коту, вероятно даже, не ведомо.

И вот ещё:
help.ubuntu.com: SecureDeletion[link1]
Гость (22/08/2013 00:46)   

Адепты церкви Большого Гильбертова Пространства сказали бы вам, что любое измерение над кубитом есть унитарный поворот совокупной общей системы, имеющей кубит своей составной частью, поэтому коллапса не существует, а информация о состоянии кубита всегда остаётся в совокупной системе. Более того, унитарное преобразование полностью обратимо, что, в частности, лежит в основе квантовых вычислений.


Только корпускулами, а не карпускулами, Ломоносов.
Гость (22/08/2013 01:26)   
Так кот Шрёденгера жив или мёртв?

По Вашему, если она обратима, то хоп, мы открыли ящик – он мёртв!

Закрыли..., подождали..., открыли – Жив!

Закрыли...
Гость (22/08/2013 06:21)   

Если подобрать эволцию правильным образом, можно и такого добиться, но это не самое интересное. К примеру, можно ускорять или замедлять течение времени у вашего «кота». Его можно сделать фактически бессмертным.

Напомнило вот это[link2], но там вроде смысл другой.
Гость (22/08/2013 13:10)   
Изучите вот эту статью: Аннигиляция[link3]
Гость (22/08/2013 23:22)   
Лучше вы эту[link4], раз вам так хочется поговорить об измерениях.
Гость (23/08/2013 17:03)   
исчезает ли в черных дырах информация
Информация не имеет смысла без носителя информации. Опишите свойства черной дыры и вам ответят на вопрос.
Гость (23/08/2013 23:53)   

Что есть смысл? Что есть информация? Что есть носитель? Сознание? Откуда берется, куда уходит, что собой представляет? Степень связности, взаимного влияния? Носитель без информации имеет смысл?
Пространство и время? Гравитация? Энергия? Эфир? Ноосфера? Можно сколь угодно долго приводить ссылки, это не отменит значение того факта, что все, что человек называет информацией, носителями её и т. д. субъективно в меру трактовки и понимания объективных явлений самим человеком. На очередном вопросе человек затыкается и смотрит на вопрошающего с изумлением, непониманием, досадой или пр. Почему реки текут? Гравитация и круговорот воды в природе? Почему гравитация? Почему? Что первично? Почему оно есть и действует так, что есть первопричина? Что было "до"? Что будет "после"? Через еще n "почему" ответа у человека не будет. Вернее будет — не знаю.
Почему ток течет по проводам? Что такое заряд? Дало (человечество) штампы, определения, условные обозначения, величины и описание на уровне своего понимания и восприятия. Понимания от этого если и прибавилось, то незначительно. На уровне, машина едет, потому что есть колеса, а мотор их крутит.

Иван Кузьмич ушел из жизни в почтенном возрасте и унес в могилу много секретов изготовления Х. Значит ли это, что информация об изготовлении Х бесследно исчезла? Полагаю, что нет. Хотя носитель информации более не присутствует в этом мире.
Гость (24/08/2013 18:47)   
Иван Кузьмич – носитель информации.
— unknown (24/08/2013 20:01)   
Всё уже давно разложено по полочкам и конкретизировано[link5] — формальная теория информации, информация в физике, в семиотике, в психологии и пр. Любое определение всегда состоит из ограничений области применения, которые накладываются и на собранную из этих определений модель.
Гость (25/08/2013 01:19)   

Некто Ферма доказал одно утверждение, но потом умер, унеся с собой в могилу его доказательство. Стало ли после смерти Ферма его утверждение неверным?

Как недавно писали на хабре, в числе π содержится вообще вся возможная информация, достаточно только offset задать. Идеальный /dev/random тоже таким свойством обладает — достаточно только сделать нужное количество запросов к нему. Так можно смотреть ещё не вышедшие фильмы, например.

Все рецепты изготовления всего, что угодно, номинально следуют из законов природы. Если начать считать, что тот конкретный тип законов, который реализуется в нашем мире — информация о нём, то да, всё из неё будет следовать, только вот осмысленно ли такое использование слова «информация» для работы с оной? Фричить на форумах проще, чем включать голову, не спорю.
— unknown (25/08/2013 22:45)   
Идеальный /dev/random тоже таким свойством обладает — достаточно только сделать нужное количество запросов к нему. Так можно смотреть ещё не вышедшие фильмы, например.

При условии, что ресурсы вселенной (не говоря о ресурсах пользователя) конечны, то можно так и не преодолеть нужный порог запросов.
Гость (25/08/2013 23:07)   

А это не важно. Главное — что гипотетически «информация есть» и может быть (гипотетически же) извлечена, что позволяет строить фрические теории и делать далеко идущие выводы.
Гость (26/08/2013 02:27)   

Не сравнивайте Ферма с Иваном Кузьмичем.
Какая логическая связь? Со смертью Ферма его утверждение исчезает? Смерть Ферма меняет структуру его прижизненного утверждения, почему оно должно становиться неверным? Вопрос об исчезновении информации.


Как недавно показывали у Малахова


Не менее осмысленно, чем вопрос с последующим обсуждением об исчезновении её, информации, в чёрных дырах в контексте идеального шифра.


А зря не спорите. Особенно, с учетом того, что в большинстве своих постов вы именно этим занимаетесь, спорами.
Перестанем же фричить. И поговорим о пертурбациях, происходящих с инфой в белых дырах!?


Не зря в своё время восьмерку повернули на 90 градусов.
Гость (26/08/2013 09:31)   

Информация о том, верна ли теорема, умирает вместе с носителем этой информации, всё по-вашему. Пока кто-нибудь другой не получит эту информацию заново из астрала, можно считать, что в мире её нет.

Гость (26/08/2013 02:27), вам никогда не приходило в голову, что вы пишете на форумах голимый унылый тупняк, засоряющий эфир?
— unknown (26/08/2013 09:50, исправлен 26/08/2013 09:51)   

А затем ввели в математику конечные объекты. И стали их изучать отдельно в дискретной математике. Которая явно ближе к криптографии и Computer Science, чем матан с бесконечностями и непрерывностями.

— юзер (26/08/2013 10:07)   
Шрёдингерскому коту, вероятно даже, не ведомо.

Не силен в этом вопросе, но любопытно, поэтому хотел бы уточнить, если кто понимает. Если я правильно понимаю, то состояние кота объективно неопределенно, т.е. даже сам кот субъективно без внешнего измерения (воздействия) не может знать (узнать) жив он или мертв??
зы. прошу не пинать, если вопрос туповат)
— unknown (26/08/2013 10:30, исправлен 26/08/2013 10:31)   

Если считать, что сама частица (или квантовая система) не может быть наблюдателем для самой себя, или как-то определять своё состояние без участия во внешних взаимодействиях, то скорее всего да.


Хотя, не силён в интерпретациях квантмеха. Прошу не пинать, если ответ туповат)

Гость (26/08/2013 11:34)   

Так говорить — misuse терминологии. Состояние всегда определённо:

| кот 〉 = α | жив 〉 + β | мёртв 〉

Чем это состояние неопределённо? Состояние | кот 〉 вполне конкретное при любых α и β. То, что вы хотите сказать, назвается не «неопределённым», а «находящимся в суперпозиции состояний | жив 〉 и | мёртв 〉». Ещё говорят, что кот не находится в собственном состоянии оператора, соответствующего измерению «жив или мёртв» (для простоты считаем, что «жив» и «мёртв» ортогональны друг другу: 〈 жив | мёртв 〉 = 0, хотя для состояния «кот» это верно только приближённо[link6]).


Слово «знать» неприменимо по отношению к квантовым частицам. Если под котом подразумевать составную скоррелированную систему, то поведение одних её частей будет зависеть от состояния дургих, но и тут слову «знать» не придать корректный смысл. Единственный объект, который может «знать» — это классический прибор, и ему, чтобы узнать, надо произвести измерение над системой.


Если под наблюдением понимать измерение, то да. Более того, квантовая система не может сама вызывать измерение себя классическим прибором. Т.е. нельзя, например, сделать так, чтобы измерение происходило тогда, когда | ψ ( t ) 〉 = | ψ' 〉, где | ψ' 〉 — какое-то заданное состояние, достигаемое системой в процессе эволюции. Если бы указанное правило не выполнялось, мы бы получили singalling (передачу информации быстрее скорости света).

P.S. Не силён в понимании того, что подразумевается обычными людьми под словами «квантмех» и «кот Шрёдингера». Прошу не пинать, если разъяснение невразумительно.
— юзер (26/08/2013 11:43)   
Если считать, что сама частица (или квантовая система) не может быть наблюдателем для самой себя, или как-то определять своё состояние без участия во внешних взаимодействиях, то скорее всего да.

Как я понимаю, быть наблюдателем можно только относительно чего-то внешнего (т.е. относительно других объектов, точек отсчета, систем и т.п.). Получается, что получить какую-либо информацию (о себе и других объектах) без взаимодествия с дргуими объектами, объект сам по себе не в состоянии, и как я понимаю информация – это продукт этого взаимодействия. С другой стороны мне не понятно, почему состояние кота неопределенно в принципе. Понятно, что оно не определенно для других объектов, до тех пор пока они не проведут измерения, но мой мозг отказывается понимать, что характеристики объекта у объекта отсутствуют, до тех пор, пока их не измерит кто-то еще..
— юзер (26/08/2013 11:52)   
Чем это состояние неопределённо? Состояние | кот 〉 вполне конкретное при любых α и β. То, что вы хотите сказать, назвается не «неопределённым», а «находящимся в суперпозиции состояний | жив 〉 и | мёртв 〉». Ещё говорят, что кот не находится в собственном состоянии оператора, соответствующего измерению «жив или мёртв» (для простоты считаем, что «жив» и «мёртв» ортогональны друг другу: 〈 жив | мёртв 〉 = 0, хотя для состояния «кот» это верно только приближённо).


Т.е. находится "в суперпозиции состояний | жив 〉 и | мёртв" для внешних наблюдателей, которые не производили измерения его состояния? А по факту состояние кота "α | жив 〉 или β | мёртв" определено и не зависит от внешних наблюдений и взаимодействий?
— юзер (26/08/2013 11:58)   
С другой стороны, характеристики: скорость, импульс, жив/мертв – это по смыслу субъективные признаки, которые привязываются к объекту субъектом измерения в меру своего понимания происходящего и в зависимости от выбранной точки отсчета. Т.е. сам объект может может быть "жив" в одной системе координат и одновременно "мертв" в другой?
— unknown (26/08/2013 15:52, исправлен 26/08/2013 15:57)   

Матчасть есть в вики:


https://en.wikipedia.org/wiki/Quantum_superposition
https://en.wikipedia.org/wiki/.....of_quantum_mechanics[link7]


Про интерпретации кота есть в самой статье о нём.

— юзер (26/08/2013 17:08)   
Про интерпретации кота есть в самой статье о нём.

Там вроде только варианты интерпретаций, когда же кот на самом деле перестаёт существовать как смешение двух состояний и выбирает одно конкретное (или в момент наблюдения, или в какой-то другой момент). Т.е. однозначного мнения на этот счет вроде нет.
— unknown (26/08/2013 17:53)   
Есть же ещё многомировая интерпретация. Кот существует в параллельных вселенных во всех возможных состояниях. Хотя эту интерпретацию критикуют за ненаучность: подтвердить или опровергнуть её нельзя.
Гость (27/08/2013 01:00)   

Ещё раз: состояние кота в вашем смысле — единичный вектор на плоскости, т.е. вполне определённый объект. Из того, что этот вектор не совпадает с одной из координатных осей в каком-то конкретном базисе (ось Ox — «жив», ось Oy — «мёртв»), ещё не следует ничего катастрофичного. Вас не удивляет то, что на плоскости есть бесконечное множество направлений, а базисных векторов всего два при каком-то фиксированном базисе?


Это расхожее мнение из глупых старых книжек, восходящее своими корнями в дискуссии 20-30-ых годов прошлого века. Уже давно ни один вменяемый специалист так не считает.


Я бы сказал, что не для внешних наблюдателей, а вообще для всех.


Измерение меняет систему. Фон Ноймановское измерение — это проектор на базисные вектора оператора той физической величины, которую вы измеряете. Помимо ортогональных состояний | жив 〉 и | мёртв 〉 можно рассмотреть другие:

| кот+ 〉 = ( | мёртв 〉 + | жив 〉 ) / √2

| кот- 〉 = ( | мёртв 〉 – | жив 〉 ) / √2

Чем для вас новые вектора | кот+ 〉 и | кот- 〉 хуже? Теперь вообразите, что есть измерение (оператор, физическая величина), для которого состояния | кот+ 〉 и | кот+ 〉 собственные, тогда измеряя систему | кот+ 〉 мы узнаём достоверно, что это | кот+ 〉, а не | кот- 〉, при этом состояние системы после измерения остаётся тем же.


Если на систему не производится воздействия, то с чего ей меняться? Если речь идёт о связанных состояниях (частица в яме), то там возможны два варианта: либо это стационарное состояние с определённой энергией, соотоветствующей собственному состоянию гамльтониана — оператора энергии системы, либо нет. Если случай первый, то | ψ ( t ) 〉 получает периодический фазовый множитель, который зависит от времени (см. стационарное и нестационарное уравнения Шрёдингера). Обычно этот множитель никому не нужен и легко вычисляется, поэтому можно условно считать, что состояние системы не меняется. Во втором случае каждая компонента вектора состояния (его всегда можно разложить по собственным векторам гамильтониана) будет вести себя так, как в первом случае, а потому весь вектор в целом будет меняться, энергия не будет сохраняться. Пример к первому случаю — стационарные «орбиты» электронов в атомах. Они себе там существуют спокойно и на ядро не падают, если находятся на стационарных уровнях. Вы эти уровни можете увидеть почти вживую по спектрам излучения, например, газовых лампочек, дискретность там налицо.


Что вы называете системой координат в квантовой механике? Я могу обозвать вектора | кот+ 〉 и | кот- 〉 новыми | жив 〉 и | мёртв 〉. Например, пусть будет «жив» — это 1, а «мёртв» — это 0. Понятно же, что я волен выбирать за 0 и 1 любые два базисных ортогональных вектора. В релятивистике всё может быть веселей,* но я не буду в неё углубляться, т.к. не разбираюсь в предмете.


В момент фон Ноймановского измерения. Что такое, по-вашему, «наблюдение»? В квантмехе это взаимодействие «квантовой системы» с «классическим прибором». За классическим прибором не обязательно должен стоять какой-то осмысленный наблюдатель с двумя руками и ногами. Когда каждая из миллиардов молекул газа сталкивается со стенкой сосуда, тоже происходит наблюдение измерение над молекулой, её состояние коллапсирует (было состояние с неопределённой координатой до столкновения, а в момент столкновения стало с определённой).


На уровне номинальной теории этот вопрос давно решён и исчерпан. Можно лезть в глубь и дебри, пытаясь, например, отказаться от понятия классического прибора, чтобы решить проблему измерений, но это скорее попытка построить другую теорию, не обязательно совместимую с квантовой механикой.


Интерпретации — не теории, это просто язык, которым пользуются люди для связки формул в нечто человеческое, поэтому интерпретацию нельзя ни подтвердить, ни опровергнуть — это просто определения, т.е., что и как мы будем называть. Кто-то бы мог сказать, что, к примеру, измерение в копенгагенской интерпретации не вполне соответствует классическому измерению и имеет уникальные свойства. Почему, тем не менее, мы называем это именно измерением, а не операцией X? Да, после операции X мы получаем знание о значении «физической величины», поэтому некоторый параллелизм есть, и термин резонный, но это не снимает вопроса. Само понятие физической величины — это тоже формальный термин с вполне формальным смыслом, как и «классический прибор». И к каждому из этих терминов можно прикопаться при большом желании. С другой стороны, сторонники копенгагенской интерпретации тоже могут прикопаться с термину «многомировая вселенная», но важно, что формулы и результаты не зависят от интерпретации, теория одна. Есть такое мнение, что квантовые вычисления и прочие современные наработки популяризовали альтернативные интерпретаци.** Говорят, что среди известных специалистов есть много их сторонников.


*Например, можно сильно увеличить время жизни нестабильной частицы, разогнав её до релятивистских скоростей. Однако, с точки зрения самого объекта, движещегося с релятивистской скоростью (космический корабль, например), время в нём течёт как обычно, он не видит изменений. Как связать тут теорию относительности и квантовую механику — для меня дебри [и, похоже, не только для меня[link8]]. На самом деле, специалист по КЭДу/КТП сказал бы, что происходит в случае частных теорий, это несложно, но я это не изучал, поэтому философствовать не буду.
**Один из расхожих аргументов (приписывают Дойчу и ещё кому-то) состоит в том, что если квантовые вычисления на многокубитных системах возможны, то одной вселенной тут никак не обойтись: у 1024-ёх кубитов пространство состояний имеет размерность 21024 — это больше, чем число атомов во всей Вселенной. Однако, если компьютер вычисляет, значит, информация обо всех этих 21024 константах (координатах вектора) где-то «хранится». А где она может хранится, если в классической Вселенной, как мы её понимаем, недостаточно для этого места? Конечно, это не доказательство и вообще не обоснованный научный аргумент, это всего лишь внутренние философские рассуждения на тему, которую человечество пока не понимает.
— unknown (27/08/2013 09:54, исправлен 27/08/2013 10:51)   

Если совсем по простому, на уровне поверхностного неквалифицированного понимания, то до момента измерения состояние квантового объекта описывается волновой функцией, в которую включены все состояния "жив-мёртв", после измерения функция коллапсирует и состояние становится однозначным.


Ну устроено оно так на том уровне, к обычным макрокотам это неприменимо, а там работает именно так. Понятия "состояние", "измерение" там определены по другому, не так как здесь (на макроуровне). Попытка приписать "жив-мёртв" такому объекту — просто юмористическая аллегория.


Более конструктивно было бы увязать этот момент с началом темы: здесь есть некая связь с теорией информации. Можно использовать это знание для понимания того, как информация о состоянии существует до и после измерений и использовать для построения квантовых протоколов или вычислений. Хотя бы для начала без чёрных дыр.

— юзер (27/08/2013 12:54)   
Если совсем простыми словами как я понял, то получается, что квантовые объекты настолько малы, что наблюдать за ними не воздействуя на них на данный момент невозможно, поэтому используют некую волновую функцию (я далек от квантовой физики, но мне из описаний выше показалось, что это что-то близкое к теории вероятностей), где описаны все возможные состояния (характеристики) такого объекта. Далее, любая попытка измерения такого объекта – это как удар кувалдой по чему-то совсем маленькому, и как результат такого измерения у объекта происходят изменения (я так понимаю коллапс), которые переводят объект в какое-то определенное состояние, которое можно зафиксировать (наблюдать). Т.е. по смыслу, после такого наблюдения объект уже совсем не тот объект, который был до наблюдения, т.к. само наблюдение существенно изменило его характеристики. Получается, что пример с котом не совсем корректен, т.к. по сути мы не можем наблюдать кота без воздействия на него за все время эксперимента и описываем его состояние с точки зрения возможных вероятностей его состояния (т.е. по сути мы не знаем), а однозначную характеристику "жив/мертв" кот получает только после измерения (удара кувалдой), и у меня есть подозрение, что сдохнет он скорее всего именно от такого измерения, а не от распада атома.
В общем, получается что никакого дуализма нет, а просто на текущий момент невозможно наблюдать со стороны имеющимися приборами такие мелкие объекты.
— unknown (27/08/2013 13:00)   
Между собой эти "мелкие объекты" взаимодействуют по таким же законам.
— unknown (27/08/2013 17:06)   
Кстати, spinore что-то говорил о бомовской теории. Вот вроде неплохой научпоп на тему[link9].
Гость (28/08/2013 04:53)   
Очередной раз понадеялся, что firefox не упадёт (падения стали реже на новых версиях), не сделал бэкап, но перед самой отправкой поста он упал. Огромный текст утерян для человечества бесследно, увы. Заново расписывать не буду, скажу только кратко: статья по ссылке — приукрашенное сектакнство, щедро приправленное чушью. Я сегодня посмотрел диссер на эту же тему (онлайн-версии нет), и там примерно то же: философии много, результатов ноль. Теория одна, а альтернативность интерпретации сама по себе ни к чему не приводит.
— unknown (28/08/2013 09:43)   
Жалко, набивайте посты в оффлайне. Бывает браузер не падает, а временно перестаёт работать сайт и отправленный пост не сохраняется. Или пропадает при постинге через Tor, если TBB не может корректно сконнектиться через него.
Гость (29/08/2013 01:40)   

В этом случае F5 часто помогает[link10].


Часто трудно предсказать длину поста заранее. Думаешь, что там немного получится, а оно всего помаленьку — и уже становится прилично. Если вовремя не сделал бэкап, это риск. С копированием текстов в бэкап тоже есть много проблем. Например, иксовый буфер не позволяет копировать длинные тексты в терминал, а графические редакторы я не привык держать инсталлированными. Короче, приходится копировать частями. Правда, обратно из текстового файла потом можно вставить и сразу оптом, если открыть файл в браузере и оттуда сделать ctrl+a ctrl+c.

Редактирование в текстовом редакторе хоть и удобное, но не сделать предпросмотр, не сделать «всё на одной строке» (в текстовых редакторах обычно принято делать автоперенос, но потом такой текст будет паршиво смотреться на форуме). Если отключить автоперенос, летят хоткеи по навигации, или я просто не умею правильно это делать. В общем, приходится набирать текст как есть с автопереносами, потом превращать поабзацно через gqap в одну строку, потом убирать лишние переносы строк, и только после этого открывать текст в браузере. Получается много действий, на которые идёшь, только если заранее знаешь, что текст будет длинный.
— unknown (29/08/2013 10:52, исправлен 29/08/2013 11:03)   

Может сделаете исключение для gvim? Чтобы информация в следующий раз не улетела в чёрную дыру :)

— юзер (30/08/2013 10:31)   
статья по ссылке — приукрашенное сектакнство, щедро приправленное чушью

За что же вы так Бома) Давайте в примере с котом представим немного усложненную версию. В ящике с котом и адской машиной есть наблюдатель (человек), который постоянно отслеживает состояние кота не влияя на систему. И есть внешний наблюдетель, который не знает что с котом, пока не откроет ящик. В принципе в моем понимании волновая функция действительна и одинакова для обоих. Но в первом случае получается, что она непрерывно коллапсирует пока ведется наблюдение за котом человеком в ящике, а во втором случае она сколлапсирует в момент, когда человек заглянет в ящик. Или все намного сложнее, и так рассуждать вообще не правильно, не научно и еще более сектантство и чушь)?
— unknown (30/08/2013 11:46, исправлен 30/08/2013 11:55)   

Человек в противогазе, наблюдающий за котом, составляет с ним одну макросистему, на которую влияет квантовое событие — ведь они друг от друга информационно неизолированы. Поскольку ящик изолирован от внешнего наблюдателя, то внутренний наблюдатель за макросистемой (частью макросистемы, которой является он сам) ничего наружу передать не может. И в судьбе кота, скорее всего, никак не участвует. Роль наблюдателя здесь как-то преувеличена. А то так и кота можно считать наблюдателем — если он успеет почуствовать, что ему плохеет. В попытке натянуть смысл через многомировую интерпретацию выглядело бы проще: живой кот остался в одном мире, умерший в другом. Внутренний наблюдатель бы на это никак не повлиял. А поскольку основные признанные интерпретации вроде как должны давать одинаковый результат, то и по-копенгагенски должно быть, наверное, также, только с другим объяснением.


Вообще, надо сначала смотреть математику процесса, затем искать её физический смысл и только в последнюю очередь строить интерпретации и упражняться в парадоксах. А то без знания матчасти можно много чего нафилософствовать.


По квантовой информации и её парадоксальным на первый взгляд свойствам есть набор теорий, исследований, публикаций и пр., но там всё естественно выводится математически.

Гость (01/09/2013 07:19)   

Железо одно, а альтернативность редакторов сама по себе ни к чему не приводит ;-)
Гость (12/02/2014 05:30)   

Некоторые ключевые моменты со слайдов «Quantum information and black holes»[link11] от Прескилла[link12], свежак:*
  1. Запрет на клонирование квантовых состояний в окрестности чёрных дыр, по всей видимости, может временно нарушаться.
  2. Временно нарушаться может и моногамия запутанности.**
  3. То, что чёрная дыра — могила для информации, может оказаться преувеличением.
  4. Сингулярность потенциала под вопросом. Скорей всего, там есть некоторое конечное дно вместо бесконечной сингулярности (см. стр. 7 и 29).
  5. В основе геометрии пространства-времени может лежать запутанность (см. стр. 41 и 51).
На стр. 32 лол.

Как Алиса и Боб могут заниматься любовью в окрестности чёрных дыр:

Love in a wormhole throat

Alice and Bob are in different galaxies, but each lives near a black hole, and their black holes are connected by a wormhole. If both jump into their black holes, they can enjoy each other's company for a while before meeting a tragic end. // Стр. 42

Чёрные дыры Алисы и Боба соединены червоточиной. Если каждый из них прыгнет в свою чёрную дыру, они смогут некоторое время наслаждаться друг другом, пока их не постигнет трагический конец кажется, я не туда пишу.

Намёки на криптографию(?):

The decoder solves a QSZK[link13]-hard problem, and hence presumably requires time exponential in the entropy of the remaining black hole.

Aaronson: if the decoding is easy, then no injective 1-way functions are secure against quantum attacks. // Стр. 49

Ключевая мысль в заключении:

— We are trying simultaneously to determine both what the theory predicts and what the theory is, without guidance from experiment. Are we smart enough to figure it out? (I don't see why not ...) // Стр. 51

Т.е. «мы пытаемся одновременно определить каковы предсказания теории и какова сама теория, при этом не оперируя экспериментальными данными». С одной стороны, чёрные дыры — самое простое объяснение тому, что должно происходить со звёздами в результате эволюции, если их масса превышает заданную. К тому же, это как-то натягивается на то, что видно в небе при наблюдении через телескопы. С другой стороны, описание этих объектов очень плохо укладывается в рамки той физики, какую мы знаем. Нам бы хотелось узнать, какая физики там на самом деле, и заодно определить границы применимости нашей земной физики, но мы лишены возможности проводить эксперименты, причём это не столько ограничение текущих технологий, сколько принципиальная особенность наших временных, пространственных и энергетических масштабов. Однако, любопытство требует, поэтому остаётся лишь схоластика с мысленными экспериментами, единственная «верификация» которой — красота и естественность перехода той общей нами воображаемой физики в нашу частную, в валидности которой мы уверены.

P.S. Предыдущий пост про работу с информацией в окрестности чёрных дыр был здесь[link14].


*Прескилл — один из отцов-основателей квантовой теории информации, который также вложился в космологию. Сейчас он директор Института Квантовой Информации в КалТехе — дочки АНБ[link15] — там же обретается и Китаев. Оттуда вышло множество других теперь всюду известных фамилий.
**Моногамия запутанности — свойство запутанности, состоящее в том, что две максимально запутанные друг с другом стороны не могут быть запутаны с кем-то ещё. Это свойство активно используется в системах безопасности, построенных на нелокальности (приборонезависимость, DI).
— unknown (12/02/2014 15:11)   
Запомнил название «квантовые информационисты». Мало того, что есть понимание того, как физика тесно связана с теорией информации, есть, так понимаю, ещё и те, кто на этом специализируется и изучает все теории именно с этой точки зрения.
Гость (12/02/2014 19:37)   
Ну хоть ктото думаит о нажей жызни.
— unknown (13/02/2014 09:40)   
Как-то не верится, что теорию могут объявить недостаточно физичной, уже потому, что она допускает исчезновение информации в некоем объекте. Вот не может информация исчезать, якобы, в чёрной дыре, значит теорию предлагается уже подгонять под это. Значит чего-то там вылетает за горизонт событий, телепортируется через излучение Хокинга, состояния там клонируются и якобы эту информацию можно считать оттуда, откуда никто и ничто не возвращается. Ну, чтобы всё сошлось по балансу.

Т.е. информация — физична, как энергия и материя? Скоро об этом будут в учебниках писать, как о постулате?
Гость (14/02/2014 22:01)   
Т.е. информация — физична, как энергия и материя?
несомненно
Гость (14/02/2014 22:57)   
Ноосфера. Об етом еще Вернандский писал.
Гость (14/02/2014 22:59)   
А где вчерашний пост про информацию, черные дыры, горизонты и прочее? Там было написано, что имеет значение поверхность, а не объем, я не успел все прочесть... не нашел в других темах, удалили что ли?
Гость (15/02/2014 01:41)   

Удалили.[link16] Оригинал тут[link17]

Пока что голографический принцип — это крайне оригинальная математическая конструкция, созданная для гипотетической Вселенной, существенно отличающейся от нашей, и предоставляющая теоретикам много новых возможностей в деле проведения различных расчётов.



Вскрытие показало, что пациент умер от вскрытия. :)
Гость (18/02/2014 01:50)   

Можно считать, что информационное описание произвольного объекта (носителя информации) — одно из возможных его (в общем случае, приближённых) описаний, таких как термодинамическое, статистическое, механическое или электротехническое.


Как бы да, но даже в элементарной теории информации вы тут же столкнётесь с непрерывностями и бесконечностями, и всё из-за того, что вероятность — непрерывная величина, а всякие функции от вероятностей/распределений любят стремиться к бесконечности в ряде точек.


Понятие научности/ненаучности к интерпретациям неприменимо, как уже было сказано. Мой начальник, например, придерживается эвереттовской, а не копенгагенской интерпретации. Какой-то раз зашёл разговор на эту тему, и мы друг друга не поняли. Споры об интерпретациях почти что религиозные. Эвереттовцы считают, что постулат о коллапсе эстетически звучит очень грубо и плохо объясняет суть происходящего, он слишком искуственный.

Представьте себе классический прибор, который ещё не настолько классический, чтобы полностью потерять всю квантовость. Пусть этот прибор взаимодействует с квантовой системой. Допустим, мы описываем обе системы чисто квантовым образом, как будто большая квантовая система взаимодействует с маленькой квантовой. В этом случае можно как бы забыть про термин «классический прибор» и посмотреть, что будет происходить с системами с точки зрения чисто квантовой механики. Вроде бы есть модельные теоретические системы и эксперименты, где такое пытаются изучать. Вроде так можно наблюдать (объяснить), например, декогеренцию и вымирание недиагональных членов в матрице плотности. Однако, по всем правилам квантовой механики, после взаимодействия массивная система, произведшая «измерение», будет находиться в суперпозиции. Грубо говоря, с одной вероятностью она должна будет показать одну информацию о маленькой системе, а с другой вероятностью — другую. И вы эту суперпозицию, оставаясь в рамках механики без традиционных «классических приборов», не убьёте никак. Почему же тогда на практике при наблюдениях мы видим только какое-то одно конкретное значение, а не их суперпозицию? Почему мы наблюдаем суперпозицию маленьких частиц, а суперпозицию больших классических приборов не видим?

Ответ как бы в том, что если не вылазить за пределы «чисто квантового» мира, то всё хорошо: измерений нет, классических приборов нет, коллапса нет, всю эволюцию можно считать унитарной, всё гладко, всё хорошо. А раз так, значит на «нижнем уровне» выбора одного варианта из нескольких никогда не происходит, все и всегда (в общем случае) остаются в суперпозиции. Тут приходит Эверетт и говорит: пусть всё так и есть на самом деле, а классические приборы — ну, это наше несовершенство, что мы не можем видеть мир таким, каков он есть, неспособы воспринимать суперпозицию, хотя на самом деле она всегда существует. А если существует, то где? В других мирах. При каждом измерении мир расщепляется на n Вселенных, в каждой из которых наблюдается только один из вариантов измерений, а вы с какой-то вероятностью (вычислимой по правилам квантовой механики) после каждого измерения попадаете в один из таких миров.

Как вы поняли, чуда не произошло. Эверетт так и не объяснил, почему при измерении мы видим только один вариант из суперпозиции, хотя внутри квантовой механики никаких причин для такого поведения нет. Тем не менее, Эверетт попытался сделать интерпретацию квантовой механики более квантовой. Если считать, что все мы с вами — кубиты, и взаимодействуем друг с другом по правилам квантовой механики, то такое объянение нам бы показалось, наверное, более мягким/естественным. Т.е. коллапса нет (ну просто потому, что в чистой квантовой механике его действительно нет), зато есть много миров для тех, кто не способен увидеть тонкость истиного квантового мира. Получается, что много миров — это костыль для несовершенных людей, живущих в классической физике, а на самом мире мир один и суперпозиция (в общем случае) есть всегда.

А сторонники копенгагенской интерпретации не мудрствуют на этот счёт и принимают коллапс таким, какой он есть. Я клоню к тому, что за любой интерпретацией стоит некоторая своя картинка со своим физическим смыслом, но выбор картинки — вопрос чисто религиозный, вкусовой. Ни одна интерпретация не отвечает по сути на большее число вопросов, чем другая. На формулах и предсказаниях результатов выбор интерпретации тоже не сказывается, это просто язык для облегчения коммуникации между людьми.

Не стоит много думать над этим, т.к. можно сломать мозг. Проще всего выбрать конструктивную точку зрения: признать своё несовершенство и не напяливать классическую логику на более общую, квантовую. Аксиоматика должна быть усвоена на уровне базово ощущаемых интуитивно воспринимаемых неопределяемых понятий. Её нельзя выразить через классические понятия, потому что последние слишком узки для этого.


Точнее, энергия не будет сохраняться в том смысле, что в разные моменты времени можно будет получать разные значения, но в среднем по времени значение энергии будет тем же. В замкнутой системе энергия не может поменяться.


Вкорне неверное утверждение. Из законов квантовой механики следует множество вплоне проверяемых экспериментальных фактов, явлений и своеобразной статистики. Работы последних десятилетий (а особенно бурный всплеск интереса после 2000-ых) показывают, что даже самые невинные мельчайшие модификации к этим постулатам немедленно приведут к несовместимости с экспериментальными данными. Одна из таких модификаций — попытка уточнения теории (т.е. сказать, что мы не можем её видеть, но такая теория есть и такая информация в квантовых системах есть). Это срач про так называемые скрытые переменные. Он считается в целом закрытым неравенствами Белла. Хинт: нелокальность.

Если коротко, то не «невозможно», а «вы упираетесь в квантовый предел на точность измерений», про который вы знаете, что он — не недостаток ваших знаний или имеющихся технологий, а принципиальное ограничение со стороны природы (по крайней мере, ни одна из существующих подтверждённых теорий его не отрицает, очередь осталась только за квантовой гравитацией).


В целом — да. Но не все характеристики меняются при всех измерениях, поэтому говорят только об изменении состояния объекта, а не самого объекта. Например, ион в ловушке после измерения не перестаёт быть ионом. Кроме того, имея ансамбль идентичных частиц и проводя измерения над ними, по набранной статистике можно установить, какие были состояния у этих частиц до измерения.


Не по сути мы не знаем, а теория говорит о том, что более детально это не определено в принципе. Квантмех — принципиально вероятностная теория, в отличие от статистики. Она не может быть уточнена. Любое её уточнение будет противоречить эксперименту. Классический мир с детерминистичными теориями — лишь частный случай общего мира. Чем быстрее вы это осознаете, тем меньше будет противоречий в голове.


Зная заранее (из расчётов), какое у частицы состояние (с какими коэффициентами взята суперпозиция), можно унитарно повернуть систему так, что у неё состояние будет совпадать с одним из базисных векторов для измерения "жив/мёртв". В этом случае без всяких измерений может будет сказать, что вы получите конкретный ответ вполне детерминистичным образом, если решите произвести измерение. Оказаться в состоянии, соответствующем базисному вектору оператора измерения, система может множеством вариантов (то же справедливо и про суперпозицию относительно этих же векторов). «Удар кувалдой» тут непринципиален. Оказаться в таком типе состояния из-за измерения — частный случай.


Устаревший термин, не пользуйтесь им. Квантовое состояние не является ни классической частицей, ни классической волной, ни их (какой-то классической) смесью. Это нечто третье, что не имеет аналогов в обычном классическом мире. Привыкайте к этому, считайте такой объект базовым неопределяемым понятием, математически оно — просто вектор в векторном пространстве.
Гость (18/02/2014 03:59)   

Если вы будете воспринимать сказанное про кота буквально, получится чушь. Шрёдингер привёл этот пример лишь для того, чтобы показать абсурдность законов квантовой механики с точки зрения классического мира. Unknown в целом вам уже ответил. Непредсказуемо для кота и всех остальных наблюдателей атом в какой-то момент распадётся и кот помрёт. Можете наблюдать кота или не наблюдать, это ни на что не повлияет. Вместо наблюения распадания атома можно поступить проще: написать программу, которая в какой-то случайно выбранный момемент времени (с определённым распределением вероятности) выдаст сигнал, приводящий к раскрытию ампулы с ядом.

Если же всё вернуть на свои места, и вспомнить, что кот — прообраз квантовой частицы, то ответ на ваш вопрос даётся квантовым эффектом Зенона[link24]. В простейшем случае можно считать, что там сидит какой-то вектор состояния, который из-за эволюции поворачивается от одного базисного вектора в сторону другого. Когда вы производите измерение, вектор состояния отскакивает и опять выстраивается коллниеарно базисному, а потом снова начинает «отпадать». Если вы достаточно часто его «стукаете», то система может никогда не распасться. Кажется (могу наврать), что-то подобное используется для борьбы с декогеренцией — применение определённого измерения, которое будет время от времени возвращать систему к исходному состоянию. Короткий ответ — система не сколлапсирует, и не важно при этом, кто её наблюдает, внося возмущения в её состояние. Достаточно того, что её наблюдает какой-то классический прибор (а с квантового наблюдения над квантовой системой вы всё равно никакой информации не получите).


Эвереттовская интерпретация говорит, что все возможные события всегда происходят, но после измерения вы попадаете в тот мир, где произошло именно то, что показал вам прибор. В какой мир вы падаете, определяется вероятностями получения конкретных результатов при измерении. Копенгагенская интерпретация говорит, что миров нет, и всё на самом деле «коллапсирует» при измерении (происходит выбор только одного варианта из нескольких), при этом прибор показывает только один результат, но какой результат будет показан, выбирается согласно тем же самым вероятностям. Соответственно, никаких различий в наблюдаемом нет. Какую бы интерпретацию вы ни применяли (т.е., какие бы слова вы ни произносили), результаты измерений будут ровно теми же.


+1.

Многие кажущиеся парадоксы — естественное следствие теории и вычислений, если знать матчасть. Например, один из постулатов гласит, что с системой проассоцированое конкретное гильбертово пространство (читайте «векторное», если по-детски). Т.е. система — это и есть, собственно, пространство. А конкретное состояние системы — вектор в этом пространстве. Как все знают из курса элементарной линейной алгебры, пространство — это группа над полем, но в группе, по определению, есть замкнутость относительно операций в группе. Если совсем пошло, то любая линейная комбинация (читай «суперпозиция») векторов пространства принадлежит пространству (если это не так, то это не пространство, а другой объект).

В пространстве существует несчётное число разных (даже ортонормированных) базисов. В этом пространстве также действуют эрмитовы операторы (альтернативно одарённые могут себе вообразить симметричную матрицу для оператора). И есть такой базовый факт из линейной алгебры, как то, что собственные вектора эрмитового оператора образуют базис, причём, не ограничивая общности, его всегда можно сделать ортонормированным. А эрмитов оператор — он и есть измерение, просто по определению (любое измерение соответствует заданию эрмитового оператора, и любой эрмитов оператор можно трактовать, как задающий измерение). Из того, что вектор состояния системы не совпадает с одним из собственных векторов какого-то эрмитового оператора, у кого-то начинается разрыв шаблона и вопли про котов, негодование, «как же так?».

Опускаясь на уровень детсада: возьмите вы обычное трёхмерное пространство, ℝ3, задайте там симметричную матрицу в каком-нибудь базисе, найдите её собственные вектора, а потом поудивляйтесь, почему произвольный наугад выбранный вектор не является одним из этих собственных. Действительно, почему? Вот везде, где состояние не является одним из собственных векторов такой матрицы, оно находится в суперпозиции (линейной комбинации) относительно этих векторов (не при этом может не быть в суперпозции относительно другой тройки векторов).

А ещё в этом пространстве можно задать унитарный оператор (читай «ортогональную матрицу, задающую поворот вектора») и действовать им на вектор состояния системы. Или просто перевести всё описание с языка операций в одном базисе на язык операций в другом. Обычно для удобства за базис выбирают собственные вектора того эрмитова оператора, который соответствует хорошо известному измерению, и с которым удобно работать.

После измерения вектор состояния системы становится собственным вектором того эрмитова оператора, который соответствует этому измерению. В случае ℝ3 таких векторов три штуки.* Какой из них будет выбран? Возьмите исходный вектор состояния системы и разложите его по этим собственным векторам. Квадраты модулей коэффициентов разложения дадут вам вероятности выбора для каждого из векторов. По конвенции все вектора состояний считаются нормированными на единицу, поэтому сумма квадратов модулей даст единицу, всё сходится.

Кстати, коэффициенты разложения одного вектора по другим задаются скалярными произведениями, поэтому скалярное произведение двух векторов состояний определяет fidelity — «близость» одного состояния к другому.

Это объяснение квантовой механики должно быть понятно любому, кто отучился на первом курсе (т.е. всем, кому сейчас больше 18-ти лет; лично мне после первого курса было 17), никаких более сложных объектов для базового понимания не требуется. Я даже ни одной формулы не написал, всё на пальцах объясняется.


*Чтоб не было совсем скучно: трёхмерное комплексное пространство (правда, не наше обычное, не конфигурационное), задаёт кутрит[link26] (например, частицу со спином 1). У неё есть три возможных результата при измерении спина: -1, 0 и 1. Помните про троичный компьютер[link27]? На кутритах можно собрать квантовый троичный компьютер:
In 2009, a ternary quantum computer was proposed which thus uses qutrits rather than qubits.
Гость (18/02/2014 07:00)   

CBH-теорема и вакцина Фукса[link28], QKD в операционных теориях и вторая квантовая революция[link29] — это всё об этом. О Фуксе нет статьи в вики, но он один из самых известных теоретиков[link30]. Насколько я помню, он сильно вложился в квантовую проверку гипотез, различение квантовых состояний и ввёл понятие «доступной информации» (см. его диссер) — это только то, с чем я лично сталкивался. В Н&Ч на него куча ссылок, т.е. теперь эти работы уже считаются классикой. Фукс пишет в своих философских статьях интересно, но у меня нет времени их сейчас прочитать, чтобы запостить выжимку. Если кому интересно, ссылки я ранее уже давал.

Глобально Фукс, думаю, прав. У него, кажется, где-то было (искал, так и не смог найти) в лоб написано что-то такое: раньше думали, что «физика — это частицы», потом что «физика — это волны», позже — что «физика — это квантовые поля», а теперь — что «физика — это информация». Смысл, как мне кажется, в следующем: изначально физика решала задачу предсказания результатов эксперимента (качественное объяснение результатов служило каркасом для теории). Но, как пишет Фукс, экспериимент, наблюдение — это лишь своего рода извлечение информации о состоянии. Можно ведь не только пассивно извлекать информацию, но и записывать её, обрабатывать, обсчитывать, решать задачи.

В некотором смысле этот переход в физике уже произошёл, когда начали ставить преимущественно не задачи «объясните нам эксперимент», а задачи «мы понимаем качественно, что происходит, но откуда взять количественную модель для описания происходящего?», «как нам сделать так, чтобы получилось вот это?». Т.е. произошёл переход от физики как описательной науки к физике как инженерии. Первый класс задач развился в область так называемого моделирования. Второй класс задач — это вопросы типа «Как создать материал с заданными свойствами?», «Как и на какой физической основе мне реализовать требуемый квантовый криптопротокол?», «Нужна ли мне в качестве подложки релятивистская теория или квантовая гравитация, если я хочу быстро решать такие-то классы задач? Помогут ли они эти общие теории мне?».

Собственно, информационное описание получается более общим и включает в себя обычное примитивное (традиционное физическое) как частный случай. Например, был изначально вопрос «как ведёт себя двухуровневая система, кубит?», а стал «можно ли на кубитах быстро решить такой-то класс задач? Сколько физических кубитов нужно для коррекции ошибок и подавления декогеренции? Можно ли осуществить такие-то и такие-то протоколы на кубитах?». Понятно, что ответы на последние вопросы требуют знания ответа на первый.

Можно ещё вспомнить то, что аппарат квантовой механики практически нигде не применялся до возникновения квантовой информатики. Даже более-менее общую трактовку квантовых состояний и операций почти никто не знал: все решали дифференциальные уравнения, находили спектр, и всё, на том конец. Никаких там вам произвольных унитарных операций, общих POVM-измерений, вопросов о близости и различимости квантовых состояний, следовых и бриллиантовых метриках, фиделити, очищениях состояний и пр. Если что-то из этого и было, то в редких математических монографиях, причём излагающих материал таким образом, что его практически никто в мире не понимал. А тут пришла информатика и всё это общее квантовомеханическое опасние вдруг сразу пригодилось, расцвело пышным цветом. До тех пор пока не ставили задачи оперирования с единичными квантовыми частицами, а работали только со статистикой, всё это было не нужно (и запутанность в том числе, хотя на неё указывал ещё Шрёдингер). ЭПР, нелокальность и неравенства Белла были интересны лишь узкому кругу лиц, занимающихся фундаментальными вопросами, вы освещение этих вопросов не найдёте ни в одном классическом курсе по квантовой механике. Это только сейчас вопросы запутанности и нелокальности стали по сути уже инженерными, т.к. понятно, где и как их планируют использовать, какие ништяки они позволят создать.

Конечно, прикладная физика останется такой, какая она есть со своими проблемами и традициями, но вот теоретическую физику придётся существенно переработать с учётом новых реалий. Старое поколение эти изменения, конечно, не примет, поэтому физика уровня ландавшица умрёт только вместе с её носителями-окаменелостями.


Исчезновение информации сродни исчезнованию её носителей, т.е. чуть ли ни нарушение закона сохранения энергии, как это можно терпеть? Весь смысл и философия теоретической физики в этом и состоит: представить всю физику, как раздел математики. В основе теоретической физики лежит небольшое число законов, из которых потом, строгим дедуктивным математическим методом, выводятся все остальные законы, теоремы, их следствия, и, потенциально, решение вообще любой задачи. Понимаете?

Я никогда не знал плохо помню классическую механику, но она, кажется, выводится примерно так (даже по Ландау): вы постулируете, что время однородно, а пространство изотропно и однородно. Это значит, что все законы механики одинаковы во всех точках пространства, никакое направление в пространстве не выделено, а время везде течёт одинаково.1 Потом вы постулируете (вроде это тоже требуется), что функция, описывающая состояние системы, не может зависеть более, чем от второй производной по координате. Т.е. она может зависеть только от координаты, скорости и времени. Этих постулатов оказывается достаточно, чтобы показать, что, с точностью до незначимого дополнительного слагаемого, есть лишь одна функция, которая удовлетворяет этим минимальным логичным физичным требованиям (т.е. всё очень жёстко, физика не допускает почти никакой свободы выбора для написания уравнений).2

Эта функция — лагранжиан системы, она удовлетворяет (дифференциальным) уравнениям лагранжа. Всё, теперь вся классическая механика у вас в руках, гипотетически с помощью уравнений Лагранжа вы можете решить любую механическую задачу, и любой механический результат уже заложен в этих уравнениях. Получается просто и красиво. Вот, собственно, цель теоретической физики — представить все области в таком изящном минималистичном виде, найти тот конец нити, тот минимальный набор фактов, который самым удачным и простым образом задаёт всю теорию.

Механика (как классическая, так и ОТО с квантами) преуспела больше всех на этом пути. Для СТО/ОТО там потребуется ещё какую-нибудь калибровочную инвариантность[link32] запостулировать относительно группы Лоренца. В термодинамике есть на то термодинамические потенциалы[link33]. В статистике в основе сидит вроде распределение Гиббса[link34] и эргодическая гипотеза[link35]. В криптографии — существование случайных оракулов и односторонних функций. Ввиду сказанного, если вы на каком-то шаге вдруг получаете утверждение, которое противоречит или всей теории или эксперименту, то дела плохи. Значит, что-то где-то глобально не так, что-то не учтено. Как в математике бьются за непротиворечивость, так и в теоретической физике.


Не информация физична, а физика информационна[link28]. ☺ В учебниках какое-нибудь непотребство обязательно напишут, если квантовая информатика придёт к успеху, но, боюсь теоретическая[link36]3 сингулярность[link37] наступит раньше. Частично это непотребство видно уже сейчас, когда 95% выработанных человечеством знаний приходится на XX'ый век, а в школьной программе их освещение занимает 5% от учебного времени. В вузах то же самое практически.


Скорее тогда «информатики». Во всяком случае, выражения «информатик-экономист» и «информатик-юрист» в русском языке есть. Школьные учителя информатики — тоже «информатики», а не «информационисты». Слово «информационисты» звучит так, как будто это какие-то неопифагорейцы, верующие в то, что всё может быть объяснено информацией. ☺


Сделал копию введения, первой главы, заключения и списка литературы. При наличии окна времени можно будет выложить. В общем, про Бома[link40] позже.


1Кстати, по теореме Нётер[link41] из этих постулатов сразу следуют все законы сохранения.
2После этой фразы, может быть, вам станет понятней, в чём идеология и надежда струнных теорий. Раз в нашем мире всё настолько жёстко для классов теорий, то есть надежда, что в более общем случае (чёрные дыры, квантовая гравитация) будет примерно так же. Т.е. общая идея — за любой реальной физической теорией стоит какой-то красивый простой изящный принцип. И как бы вы ни развивались, каким бы образом ни думали, рано или поздно вы до этого принципа доберётесь, все дороги ведут к нему, он неминуем. Это такой «Святой Грааль» который позволяет решить всё и вся, объяснить всё, проникнуть в самую глубь вещей. Чтобы до него добраться вам нужно навести множество логических стрелок, что из чего и почему следует. Когда все стрелки будут на месте, вы поглядите на этот клубок и увидите тот корень, конец и начало всего, откуда всё растёт. Вот примерно какая-то такая философия стоит за этими исследованиями. В том числе, активное исследование операционных теорий, пробив квантовой механики через логику и каузальность, дополнительные нефизичные допущения — это всё рисование вот таких стрелок. Действительно, посмотрите на Стандартную Модель. Теоретически там могло бы быть чёрти что, а реально всё красиво: всего[link42] 6 кварков, 6 лептонов, 4 бозона + Хиггс (ну, и всякие симметрии ещё). Ну или можно вспомнить про принцип наименьшего действия[link43] (строго говоря, экстремального действия), из которого выводится вся механика, начиная с классической и кончая КТП. Функциональный (фейнмановский) интеграл[link44] — он же целиком на этот принцип завязан, а без него вообще КТП не быть.
3См. абзац
Есть ещё такая мысль: наука — это пирамида. ...
Гость (18/02/2014 07:18)   

03:59-01:50 = 2ч. 09м. ≈ 2ч.
07:00-03:59 = 2ч. 59м. ≈ 3ч.

2ч.+3ч.+(≈)2.5 ≈ 8ч. Т.е. 8ч., чтобы ответить только на часть вопросов только в одном треде. Фактически полный рабочий день ночь. Дорого, блин, обходится. ☹ А думал, пара часов — и напишу всё, что хотел по поводу последних постов, во все квантовые треды. ☹
Гость (18/02/2014 07:45)   

Сколько вам платит госдеп? Это безобразие, публично, на виду у детей, научной пропагандой занимаются. Доколе!?
Гость (18/02/2014 07:59)   

85р/пост. Тем, кто участвовал в разгоне несанкционированного набега силовиков на форум, ещё обещали квартиру дать, коммуналку, и солидные премии.
Гость (18/02/2014 08:06)   

У одного депутата[link45] должна быть приёмная. Пусть сделает депутатский запрос. Он специалист по этой теме.
— unknown (18/02/2014 10:17)   
И[link46] вы эту суперпозицию, оставаясь в рамках механики без традиционных «классических приборов», не убьёте никак. Почему же тогда на практике при наблюдениях мы видим только какое-то одно конкретное значение, а не их суперпозицию?


Не помню всех интерпретаций, тем более маргинальных, но напрашивается банальный вопрос: а нельзя ли построить (или была ли уже построена) интерпретация с мерой «квантовости» в зависимости от масштаба (размера системы)? Без введения дополнительных сущностей вроде параллельных миров и коллапса? Т.е. вот на микромасштабе всё описывается таким уравнением, а на макро — вот таким. А если хотите — вот вам общее уравнение. Как в теории относительности, где можно пользоваться эйнштейновскими уравнениями, но в обыденной жизни в большинстве случаев просто не учитывается скорость света и получается частный случай более общей теории. Т.е. никакой особой интерпретации с картинками и аналогиями даже не нужно, достаточно более широкой формализации. Конечно, такой вопрос, подозреваю, вполне глупый наивный, т.к. за почти сто лет уже наверняка все варианты перебрали, в т.ч. и такой.

Проще всего выбрать конструктивную точку зрения: признать своё несовершенство и не напяливать классическую логику на более общую, квантовую. Аксиоматика должна быть усвоена на уровне базово ощущаемых интуитивно воспринимаемых неопределяемых понятий.

Именно этого бы и хотелось. Достаточно знать, что базовые свойства системы описываются вот таким-то матаппаратом, а всё остальное выводится из этого. И пока не полезут формальные противоречия, ничего менять не надо. Представлять наглядно тоже не надо: проекции и отображения многомерных объектов в нормальном состоянии сознания никто себе тоже в буквальном виде не представляет.

Тогда такой вопрос интересен, в связи с вышеизложенным. Материю можно превратить в энергию (в звёздах, реакторах, бомбах). В принципе, можно и наоборот: сталкивать что-то высокоэнергитическое и путём цепочки преобразований что-нибудь материальное насобирать из частиц в ускорителях (или представить, что там в звёздах происходит). А информацию можно как-то трансформировать напрямую? Есть какая-то формула, диаграмма, треугольник, которая связывает все эти три сущности в общем случае? Пусть хотя бы такая попсовая и не совсем верная как E=mc2. Вот куда тут информацию воткнуть?

То что материя и энергия задают предел вычислений — понятно, хотя бы через лимит Ландауэра, а информацию можно взвесить в граммах/джоулях? Или перевести биты информации в них не только чисто умозрительно-вычислительно, но и как-нибудь наглядно, с какими-нибудь измеримыми эффектами?
Гость (18/02/2014 13:27)   
Это объяснение квантовой механики должно быть понятно любому, кто отучился на первом курсе
Ты чо, издеваешъся? Я, выпускник академии МВД, не понял ни слова. Заясни ищё проще.
Гость (18/02/2014 22:20, исправлен 09/04/2014 14:55)   
Устаревший термин, не пользуйтесь им. Квантовое состояние не является ни классической частицей, ни классической волной, ни их (какой-то классической) смесью. Это нечто третье, что не имеет аналогов в обычном классическом мире. Привыкайте к этому, считайте такой объект базовым неопределяемым понятием, математически оно — просто вектор в векторном пространстве.

Какое-то загадочное это квантовое состояние. Потрогать нельзя, наблюдать нельзя, классической физикой понять нельзя. Соединить в единую систему взаимодействия с классическими телами и макро объектами (как я понимаю теория (или модель) всего) тоже нельзя. Выходит, что "проще всего выбрать конструктивную точку зрения: признать своё несовершенство и не напяливать классическую логику на более общую, квантовую".

Гость (19/02/2014 06:38)   

Чтобы так можно было сделать, нужно, полагаю, как минимум, решить проблему измерений (нерешённая проблема современной физики) и, как максимум, заменить квантовую механику детерминистичной теорией.

T[link48]he measurement problem in quantum mechanics is the unresolved problem of how (or if) wavefunction collapse occurs.
...
the Schrödinger wave equation determines the wavefunction at any later time. If observers and their measuring apparatus are themselves described by a deterministic wave function, why can we not predict precise results for measurements, but only probabilities? As a general question: How can one establish a correspondence between quantum and classical reality?

Через декогеренцию это тоже объяснить не получается:

D[link49]ecoherence does not generate actual wave function collapse. It only provides an explanation for the observance of wave function collapse, as the quantum nature of the system "leaks" into the environment. That is, components of the wavefunction are decoupled from a coherent system, and acquire phases from their immediate surroundings. A total superposition of the global or universal wavefunction still exists (and remains coherent at the global level), but its ultimate fate remains an interpretational issue. Specifically, decoherence does not attempt to explain the measurement problem. Rather, decoherence provides an explanation for the transition of the system to a mixture of states that seem to correspond to those states observers perceive. Moreover, our observation tells us that this mixture looks like a proper quantum ensemble in a measurement situation, as we observe that measurements lead to the "realization" of precisely one state in the "ensemble".

Т.е. декогеренция якобы объясняет, почему в матрице плотности вымирают недиагональные члены, превращая всё состояние в обычную суперпозицию,1 а вот из-за чего происходит выбор только одного конкретного вектора в суперпозиции, остаётся неясным.

Если отвечать на ваш вопрос буквально, то нужно сказать, что квантовая механика на макромасштабах якобы существует и действует, но ни к чему этот факт не пришить, т.к. у нас нет иного способа черпать информацию о состояниях, кроме как методом взаимодействия системы с классическим прибором. Мы можем рассчитать, что будет при взаимодействии квантовых частиц друг с другом, произвести миллион унитарных операций и т.д. и т.п., но в конечном счёте мы захотим узнать, то ли там получилось, что мы насчитали. А узнать это можно только через классический прибор. Других измерений у нас нет. Т.е. ничего о квантовом состоянии иным способом узнать нельзя, а этот способ разрушающий, ведёт к коллапсу. Тут можно ввести некие поправки, произнеся слова про POVM, но смысл в целом сохранится, поэтому не буду лезть в дебри. Грубо говоря, описание макросистем с помощью квантовой механики будет совершенно неверифицируемым. Ничего нового, помимо результатов классической физики, вы не увидите.

Масла в огонь подливает то, что вся классическая физика детерминистична, а квантовая — нет. Допустим, у нас есть какое-то уравнение эволюции, которое описывает, что происходит при взаимодействии макросистемы (классического измерительного прибора) с квантовой частицей. Но у уравнения, описывающего состояние классического объекта, есть вполне конкретное решение,2 оно детерминистично. Из-за чего состояние классического прибора может вдруг стать недетерминистичным? Более того, коллапс — это же могила для информации, это необратимая операция, хотя по отдельности как классическая механика, так и квантовая обратимы.3 Но как только вы столкнули классику с квантами, вы тут же получили необратимость. Что с этим делать? Т.е. даже на концептуальном уровне непонятно, как так называемая «проблема измерений» может быть в принципе решена.

Другая известная проблема (опсана в том же ЛЛ-III) — наличие порочного круга в логических основаниях квантовой механики. Онтологически мы понимаем, что квантмех должен быть более общей теорией, чем классика, поэтому классика должна быть выводима из квантмеха как его частный случай. Но что мы имеем на практике? Чтобы сформулировать постулаты квантовой механики, вам нужно запостулировать существование классических приборов, которые суть объекты той классической теории, которая должна следовать. А мы это закладываем в основу. Получается, что не имея классической механики, мы не можем сформулировать квантовую.

Кроме того, чтобы задать эволюцию квантовой системы, надо задать её гамильтониан, т.е. энергию системы (в любой её конфигурации) как функцию от координат и импульсов. Откуда взять гамильтониан? Ну, можно запостулировать, но тогда придётся постулировать гамильтонианы всех возможных систем, а их бесконечно много. На практике гамильтонианы крадут с классической физики, потом объявляя их в квантовой как данность. Процедура называется «квантованием». Типа, вот, была классическая система, её «проквантовали» и получили «её квантовую версию». На практике при квантовании есть некоторая свобода выбора, но обычно следуют конвенции, задаваемой правилом соответствия Вейля[link50]. У Де Госсона об этом неплохо написано[link51].

Трюк там примерно такой: вы расписываете гамильтониан, как функцию координат q и импульсов p (считаем, что функцию можно разложить в ряд Тейлора/Лорана по степеням q и p), потом каждое произведение qn pm симметриузете, чтобы q и p входили в него симметрично, после чего проставляете над ними шляпки, превращая их в операторы и . Что такое — задаётся по определению, а оператор для выражается через собственные вектора оператора , если воспользоваться производной от оператора и соотношением неопределённостей (коммутационным соотношением) [,]=i. В первой части методички[link52] эти процедуры подробно расписаны. Вот такое «квантование». И где тут независимость «более общей» квантовой теории от «более частной» классической?

Последнее время даже такая новая ересь философия появилась, что никто ни для кого не более общий, «они просто разные». Квантмех, если сильно извратиться[link53], можно описывать почти классически. Классику, если немного поизвращаться, тоже можно описывать квантово. И, вообще, там естественным образом можно насоздавать какие угодно механики, не обязательно квантовые или классические. Кроме того, про любое распределение можно будет сказать, реализуемо ли оно в классике, в квантах или в обоих теориях сразу.

На эту тему есть отдельный троллинг, которым троллят КТПшников (или струнщиков?). Просто задают им вопрос «Откуда вы взяли такой лагранжиан поля? А почему именно такой? А почему не вот этот?». Функция Лагранжа (лагранжиан) играет ту же роль, что и гамильтониан, он задаёт систему. Не для всех случаев жизни, особенно сложных и патологических, всё известно. Когда можно, Лагранжиан тянут из классики, всё как в нерелятивистских квантах. Когда нельзя, начинается философия и спекцляции, пальцевые оценки и общие рассуждения. Но, извините, разные лагранжианы дают разные решения. Хорошо, если есть экспериментальная верификация, и можно договориться, какой лагранжиан считать правильным, а если с экспериментом туго? А если там чёрные дыры? ☺ В общем, струнщикам вдвойне тяжело, потому что даже в нерелятивистских квантах не всё просто и не всё понятно.


Да, и для особо дотошных есть история физики[link54]. В неё можно заглянуть и узнать, почему и как догадались именно до таких постулатов, как их можно получить индуктивно (хотя бы в принципе) из наблюдений эксперимента.


Многие вещи в многомерных пространствах представить крайне трудно, но кое-что можно. Я когда в детстве пытался понять переход от декартовых координат к сферическим в n-мерном пространстве. Действительно, делая по аналогии, начинаешь постепенно понимать, почему оно именно так. Т.е. какие-то обрывочные геометрические образы в голове при этом возникают. Просто многие операции можно вообразить, если иметь доступ только к двум-трём векторам базиса из n базисных векторов. С другой стороны, даже такие простые объекты, как двумерное комплексное пространство уже не могу себе представить какой-то картинкой. А функции комплексного переменного хотя и кажутся совершенно абстрактными, у них, говорят, есть прозрачный наглядный физический смысл: это гидродинамика на плоскости. Представьте себе поле скоростей жидкости на плоскости, где могут быть завихрения, источкники и стоки — вот это оно и будет. Источник/сток — это дивергенция, завихрения — роторы.


Вы же совсем недавно уже задавали[link55] этот вопрос, и в меру своих знаний я на него ответил. Чтобы это звучало не так пафосно, можно слово «информация» всюду заменить на «энтропия». По ссылке есть отсылки к Ллойду, но это пока не уровень общеизвестных и общепризнанных фактов. Как вы видели из последних постов, даже с принципом Ландауэра ещё только разбираются, квантовая термодинамика в стадии зарождения, поэтому до базовых фактов ещё не добрались. Что-то фундаментальное уже известно. Например, то, что двухуровневая квантовая система не может[link56] содержать больше одного бита. Т.е. большего количества осмысленной (информационнонесущей) энтропии вы туда не засунете никак. Зная ограничения физики на пространство/плотность материи/время можно что-то пересчитать в информацию, но это будет идти речь о релятивистике, а релятивистская информатика ещё только развивается[link14]. Про информатику поверх квантовой гравитации (то, на что замахивается Ллойд) я вообще ничего не слышал. Т.е. отдельные работы есть, попытки есть, но сформировавшегося научного направления со своей школой ещё нету, не зря же одной из целей доклада Прескилла[link57] было привлечь интерес широкой публики к этой теме и вообще объяснить ей, что же может быть общего между современными струнщиками и современными «информатиками».4

Но раз вам так хочется универсальных красивых формул, могу их вам дать — это широко распиаренные в народе граница Бекенштейна[link58] и предел Бреммермана[link59]:

This value is important when designing cryptographic algorithms, as it can be used to determine the minimum size of encryption keys or hash values required to create an algorithm that could never be cracked by a brute-force search.
...
For example, a computer with the mass of the entire Earth operating at the Bremermann's limit could perform approximately 1075 mathematical computations per second. If we assume that a cryptographic key can be tested with only one operation, then a typical 128 bit key could be cracked in under 10−36 seconds. However, a 256 bit key (which is already in use in some systems) would take about two minutes to crack. Using a 512 bit key would increase the cracking time to approaching 1072 years, without increasing the time for encryption by more than a constant factor (depending on the encryption algorithms used).

Только учтите, что эти формулы родом из пока что несуществующей теории квантовой гравитации, поэтому спекулятивность этой оценки достаточно высока[link60]. Я предупредил.

Вообще, концепция, наиболее близкая к теме — цифровая физика [en[link61],ru[link62]] и, более общо, цифровая философия [en[link63],ru[link64]]. Ещё есть цифровая вероятностная физика[link65]. Поскольку я не специалист в этой теме и эти ссылки сам вижу впервые, прокомментировать лучше, чем там написано, не могу. Но, в принципе, это интересно. Даже не знал, что такое уже есть.

На тему этих парадоксов и гипервычислений есть ещё срач на тему «волновой функции Вселенной». Если представить, что вся Вселенная описывается квантовой механикой, то можно включить в систему вообще всё на свете, сделав систему замкнутой. Но у замкнутой системы нет коллапса, и эволюция её может быть только унитарной. Как результат, все действия во Вселенной становятся в такой трактовке обратимыми, хотя на практике мы эту обратимость не видим. В общем, тут целый кладезь для парадоксов, возникающих то ли из-за фатального непонимания, то ли из-за полной неприменимости квантовой механики в космических масштабах. Напомнило пресловутую тепловую смерть Вселенной[link66] — это как раз энтропийный (т.е. информационный) парадокс.


Д[link67]ирак любил выражаться точно и требовал точности от других. Однажды он выступал с докладом на семинаре. Закончив сообщение, Дирак обратился к аудитории: «Вопросы есть?». — «Я не понимаю, как вы получили это выражение», спросил один из присутствующих. «Это утверждение, а не вопрос, — ответил Дирак. — Вопросы есть?».


К сожалению проще нельзя, можно только сложнее.5 Линейная алгебра — самый простой матаппарат, опираясь на который можно положить квантмех. Базовая его часть известна школьнику (операции с векторами, интуитивное понятие пространства), в продвинутых матшколах солидный кусок вузовской линейной алгебры — вообще часть общеобразовательной программы. Более подробно это рассматривают на первых математических курсах после школы (хотя бы на уровне «кто такие матрицы, зачем они нужны, и как с ними работать»). Без матриц это объяснить не получится, если цель — не вносить существенных искажений в суть предмета. Искажённых превратных описаний того, что есть квантмех, хватает в интернете, я бы не стал писать ещё одно такое же, очередное.

Попытка №2

На самом деле, есть ещё популярное введение на языке вигнеровского представления и звёздочного квантования, но его достаточно сложно сделать полным (дать полную трактовку теории, используя только этот язык), хотя чем-то оно интуитивно проще. Смысл его в том (на примере классической механики), что состояние частицы задаётся координатой q и скоростью импульсом p. В одномерном случае это точка на координатной плоскости с осями q и p, её называют фазовой плоскостью или wwwфазовым пространством системы. Когда система эволюционирует, эта точка движется по плоскости, вырисовывая на ней какую-то кривую. Например, осциллятор без трения будет описывать эллипс, осциллятор с трением — эллипсоидную спираль, закручивающуюся к нулю, равномерное прямолинейное движение — луч, параллельный оси координат q. Математически состояние системы в любой момент времени задаётся этой точкой, что удобно записывается через wwwδ-функцию6 как Wclassic = δ(q(t)-q0) δ(p(t)-p0).

Это была классика. В квантах будет так же, только Wquant более не произведение δ-функций, а некоторое квазираспределение от q и p (т.е. была точка, а стала поверхность, какое-то распылённое облако «вероятностей» вокруг точки). В частности, интегрирование Wquant по q будет давать функцию распределния по p (и наоборот), но сама по себе функция Wquant не является совместной плотностью вероятности для q и p (это бы нарушало соотношение неопределённостей). Эту Wquant называют wwwфункцией Вигнера. Она есть полностью эквивалентный аналог вектора состояния в этой формулировке квантовой механики. По ссылке есть картинки с её графиками-поверхностями для некоторых состояний. Здесь существенно ещё то, что всё вещественно. Нет никаких комплексностей ни на каком этапе (зато есть отрицательности).

Произвольная двумерная поверхность не будет валидной функцией Вигнера, она несовместима с квантмехом. Простых способов проверить функцию на валидность тоже нет — надо либо пересчитывать её в матрицу плотности и проверять её положительность/симметричность, либо проверить бесконечное число соотношений на, кажется, средние значения некоторых моментов. Т.е., как теоретическая конструкция она полезна, но на практике не всегда удобна (хотя в каких-то практических экспериментальных задачах она удобнее всего остального). Что удобно в таком представлении — так это считать среднее от функций, зависящих от q и p, по данному состоянию. Считая функцию Вигнера как будто настоящим двумерным распределением (плотностью вероятности), можно найти среднее значение и дисперсии для любой классической физической величины, представимой в виде функции от q и p.

Есть ещё знаменитая теорема Худсона (Hudson), которая говорит о том, что если функция Вигнера чистого состояния (состояния изолированной системы) положительна, то она гауссова. Реально она, конечно, не всегда положительна. Простейший пример — первое возбуждённое состояние осциллятора, где сидит ровно один фотон энергии (состояние |n〉=|1〉). А если бы мы говорили о своём незании информации о системе, где всё классично (речь о классической статистике), то такая функция могла бы быть только положительной. Отсюда идут знаменитые речи о том, что квантовая механика соответствует, в общем случае, отрицательным вероятностям, что как бы подчёркивает её полную несовместимость с классическим миром.

Далее, понятно, что функция может зависеть от времени. Уравнение эволюции для функции Вигнера тоже есть. Потом, вместо действия операторов можно брать свёртку двух функций, что в практическом случае превращается в интегрирование произведения двух функций (нередко ещё и обобщённых, поэтому там полный атас). А вот суперпозицию двух состояний вы легко на этом языке не запишете вообще. Конечно, всегда можно пересчитать эти функции в волновые (или вектора состояния), взять их суперпозицию, а потом обратным преобразованием через взятие интеграла узнать, чему это будет соответствовать на фазовой плоскости. Но никакого более простого правила нет. Т.е. чем-то наглядности стало больше, а чем-то меньше. Вектора хотя бы образовывали хорошую математическую структуру — пространство, а функции Вигнера, как функции, ничего такого красивого не образуют. Обо всём этом можно прочитать в wwwвики.

Наконец, можно сделать по аналогии, завязавшись не на пространство q и p, а на любые другие независимые (канонические) переменные, получив другую «формулировку» квантовой механики, полностью эквивалентную всем предыдущим. Иногда вместо функции Вигнера берут wwwQ-функцию Хусими или wwwP-функцию Глаубера-Сударшана. В квантовой оптике эти представления достаточно популярны, и в квантовой информатике с непрерывными переменными (включая CV QKD) тоже. В первой части методички математика по этим функциям неплохо изложена.

Выводы квантовой механики через такие функции (традиционную волновую функцию / матрицу плотности тоже можно рассматривать как их частный случай) объединили под зонтиком «звёздочного квантования» и «звёздочного произведения». В вики оно названо wwwпроизведением Мойала. Суть в том, что любому состоянию (а также любому оператору, действующему на состояние) в звёздочной трактовке соответствует некоторый символ. А поскольку все квантовомеханические операции можно представить в виде операторных выражений, на звёздочном языке получается, что всё можно представить в виде произведения символов. В частности, функция Вигнера — это символ оператора плотности (матрицы плотности). Произведение символов обязано быть ассоциативным, но не обязано быть коммутативным. На практике, чтобы задать это произведение символов (т.е. звёздочное), надо задать конкретное ядро интегрального оператора.

Кстати, у этой звёздочной формулировки квантмеха есть смысл «расквантовывания» квантовой механики. Изначально была классика, там всё было коммутативно (числа друг с другом коммутируют). Кванты долго не могли написать потому, что понимали, что нужны какие-то некоммутирующие числа. Физики в те времена математику вообще знали плохо, и про матрицы/операторы даже не слышали. Например, Дирак ввёл свои некоммутирующие числа, назвав их q-числами (в противоположность классическим, c-числам). Потом, конечно, все поняли, что q-числа, — это просто матрицы, а если отвязаться от конкретного базиса, то какие-то операторы, но всё равно сложился шаблон: классика — коммутирующие вещи, функции, а кванты — некоммутирующие вещи, операторы. Соответственно, переход от операторов к функциям (раз это возможно в альтернативных формулировках квантмеха) соответствует как бы «деквантизации». В частности, оператор, переводящий символ в оператор, называется «квантайзером», а оператор, находящий символ оператора по самому оператору — «деквантайзером».

Пользуясь какой-то конкретной схемой звёздочного произведения можно описать как квантовую механику, так и классическую (а также классическую статистику), отличия будут только в правилах взятия произведений. Можно описывать даже какую-то третью, несуществующую, механику, если для неё хватает этого (кстати, достаточно общего) звёздочного языка. Интерес этих представлений в том, что для наблюдаемых физических величин их символы в классической и квантовой механике оказываются одними и теми же. Это первый забавный параллелизм. Второй параллелизм в том, что ядра для взятия произведений в классике и квантах оказываются очень похожими. Например, ядро в квантах точно такое же, как классическое, но домноженное на экспоненту с чем-то, имеющим вид типа коммутатора (допустим, μ1 ν2ν1 μ2). Т.е. это какой-то привет-отголосок от соотношения неопределённостей. И лишь эта маленькая поправка в виде коммутатора в экспоненте (операторов там нет, это просто вещественные параметры) превращает классическую механику в квантовую. Интересно?

Это всё про непрерывные переменные. Дискретные переменные (спин, поляризация) излагаются отдельно в обоих случаях (хоть звёздочный язык, хоть дираковский традиционный), потому что объекты дискретного пространства живут в совсем другом пространстве, не имеющем никакого отношения к нашему обычному, конфигурационному. Правда, в дираковском описании параллелизм между дискретными и непрерывными переменными самый прямой,7 а в звёздочном «квантовани» для спина надо городить что-то своё, отдельное, типа спиновых томограмм.

Когда я пытался написать обо всём этом первый раз, излагая тему связным образом (увы, так и не закончил), многих из упомянутых ссылок в википедии попросту не было, а сейчас уже почти всё там, всё упомянуто, всё расписано. Так можно немного подождать, и проблема решиться сама собой: достаточно будет кинуть линк на вики, сказав «там всё написано». Шутка.

Так понятней? Нам говорили, что если не поймём, то наденем сапоги и пойдём родину Государство защищать, поэтому все старались понять, хотя бы к моменту зачёта/экзамена.


Классический взгляд на время подразумевает существование изменений в мире. Как можно измерить время, если нет ни одного меняющегося процесса? С одной стороны, мы мерим время в изменениях систем. С другой стороны, изменения систем задают существование времени. Если во всей Вселенной перестанет меняться что бы то ни было, время, как физическая сущность, исчезнет. Математически это будет выглядеть, как исчезновение t из уравнений, задающих состояния. Т.е. была динамика, а теперь будет статика.


Вообще, хорошо рассуждать, если что-то изучал долго, постепенно, сначала индуктивно, потом дедуктивно, с разных сторон и т.д. Но есть опыт математиков, который говорит о том, что можно начинать сразу с дедуктивного метода.

Что-то потрогать можно, но не всё и не во всей полноте. Даже в классике не всё потрогаешь (радиволны и молекулы невооружённым глазом не видны; под напряжением провод или нет — мы тоже не определим, не прикоснувшись к нему). На самом деле, в первом приближении, вектора состояний все видели на школьных уроках химии: s-, p-, d- и прочие облака — это функции для плотности вероятности нахождения электрона |〈q|ψ〉|2, они соответствуют станционарным состояниям |ψ〉. Технически это сферические функции[link68]. Попытки объяснить образование молекул и взаимодействие между ними через всякую гибридизацию облаков — это попыка решить на пальцах уравнение Шрёдингера, где последнее точно задаёт, какие будут облака при заданной конфигурации в каждый момент времени.

Ремарка:
Однако, не стоит забывать про принципиальную тонкость: по |〈q|ψ〉|2 нельзя, в общем случае, восстановить |ψ〉 (иначе было бы всё совсем просто и наглядно). Разные |ψ〉 (и это принципиально) могут давать одинаковые «облака вероятностей» |〈q|ψ〉|2. Более того, даже если бы мы знали такое же облако, но ещё для импульсов, т.е. |〈p|ψ〉|2, то и это бы не помогло однозначно восстановить |ψ〉, но именно |ψ〉, а не что-то иное, задаёт состояние. В общем случае, |ψ〉 — комплексный вектор в гильбертовом пространстве, и именно его эволюцию описывает уравнение Шрёдингера.

Потом, то, что в стационаром состоянии электрон представляется лишь одним из вариантов облаков, но не чем-то промежуточным, говорит о дискретном спектре. А спектр дискретный, потому что, извините, у операторов в функциональном анализе для «локализованных» функций он дискретен. И уж, тем более, дискретен спектр у конечномерных операторов (матриц). Значения энергии, соответствующие разным облакам — собственные значения гамильтониана. В частности, квантовая химия[link69] пытается всё это рассчитать и описать формальным образом, решая уравнения.

Дискретность видна глазом из спектров: раз допустимые уровни энергии дискретны, электроны, перескакивающие с верхнего уровня на нижний, теряют всегда определённое количество энергии. И именно эту энергию (а, значит, частоту) имеет излучаемое электромагнитное поле (фотоны). Наблюдая за спектром гаозразрядных/ртутных ламп через спектрометр, всё это видно, а по частоте, на которой видны линии, можно восстановить энергии уровне в атоме, где сидят электроны. Получается вполне естественное заземление квантoв в практическую плоскость.


1Т.е. линейную комбинацию из тех векторов, которые мы в принципе можем получить после измерения; на матричном языке ещё можно сказать, что у вас матрица плотности становится одноранговой.
2Если оставить за рамками неустойчивости и некорретно поставленные (ill-posed) задачи[link70].
3Это следствие симметрии уравнений Гамильтона и Шрёдингера по отношению к обращению времени. В квантовом случае это, очевидно, проще объяснить тем, что эволюция, задаваемая уравнением Шрёдингера — унитарная операция, а унитарные операции обратимы (любой поворот обратим).
4Традиционно эти два лагеря вообще не пересекаются, хотя по квантовой информатике есть разделы во многих журналах по математической физике.
5Например, не языке теории групп, теории категорий, функционального анализа или C*-алгебр.
6Интересно, что δ-функция была введена Дираком специально для квантовой механики, хотя сейчас это часто используемый объект в много какой инженерии начиная с электроники.
7Всё одинаково, кроме того, что одни вектора — в конечномерном пространстве (спин, достаточно линейной алгебры), а другие — в бесконечномерном (CV, нужен уже функциональный анализ, если не вносить упрощения). Функциональный анализ по сути и есть линейная алгебра для того случая, когда размеры матриц стремятся к бесконечности.
— unknown (19/02/2014 10:21, исправлен 19/02/2014 10:27)   
Чтобы так можно было сделать, нужно, полагаю, как минимум, решить проблему измерений (нерешённая проблема современной физики) и, как максимум, заменить квантовую механику детерминистичной теорией.

Масла в огонь подливает то, что вся классическая физика детерминистична, а квантовая — нет.

Другая известная проблема (опсана в том же ЛЛ-III) — наличие порочного круга в логических основаниях квантовой механики. Онтологически мы понимаем, что квантмех должен быть более общей теорией, чем классика, поэтому классика должна быть выводима из квантмеха как его частный случай.

Последнее время даже такая новая ересь философия появилась, что никто ни для кого не более общий, «они просто разные».

Тогда можно предложить новую ересь философию: никто ни для кого из них двоих не общий (по крайней мере прямой и однозначной иерархии общностей нет), они все являются частными случаями какой-то более общей теории, которая должна описывать их обоих как частные случаи.


Разве что, с т.з. философии и методологии это как-то против «бритвы Оккама» получается, но если противоречий и необъяснимых фактов накопится слишком много, то придёться вводить новые сущности.


Бекенштейн и Бремерман интересны, примерно что-то такое и представлялось, спасибо. Поначалу оно всегда так выглядит — непрактично и натянуто, но вполне где-то может пригодиться.

Гость (19/02/2014 12:13)   

Этой теорией, в любом случае, думаю, должна быть квантовая гравитация теория всего. В виде сводной таблицы имеющиеся фундаментальные теории можно представить так:
МасштабСкорость
Малый(-ая)AB
Большой(-ая)CD

Здесь:
  • CB — классическая механика
  • CD — теория относительности
  • AB — нерелятивистская квантовая механика
  • AD — «теория всего»[link71].

Часть теории всего, уже построенная для всех четырёх фундаментальных взаимодействий[link72] и их произвольных комбинаций/смесей (сильного, слабого, электромагнитного и гравитационного), за исключением гравитационного,8 называется квантовой теорией поля (КТП). Саму КТП можно разбить на подразделы:
  • КТП только электромагнитного поля — КЭД (квантовая электродинамика).
  • КТП для электромагнитного и слабого взаимодействий вместе — теория электрослабого взаимодействия.

Недостающая часть теории всего (т.е. «теория всего за вычетом КТП») называется квантовой гравитацией. Часто «теорию всего» и «квантовая гравитация» используют вообще как синонимы. Считается, что теория всего из-за включения гравитации будет мало похожа на КТП, поэтому её обычно не называют КТП. Одной из основных претендентов на общую теорию всего является теория струн M-теория[link73], но она пока слишком общая, и ничего там толком посчитать не удаётся. Другой популярный кандидат — петлевая квантовая гравитация[link74], но есть и другие. Вот примерно такой расклад.


Про Бекенштейна я сам узнал недавно и чисто случайно. Бремерман и остальные цифровые физика/философия после этого элементарно находятся по ссылкам внизу страниц википедии, там все статьи друг с другом слинкованы. Тем не менее, когда читал это в предыдущие разы, почему-то не добрался до этих страниц. Видимо, там со связностью что-то не то.

Когда читал про «информационную физику» [типа ваш термин, можете его ввести в гугл9], понял, что всё на поверхности, и достаточно-то всего одного поискового запроса было, чтобы все эти ссылки найти. Подумалось, что раз вы спрашиваете, то уже гуглили и в курсе этих вики-статей, но хотите услышать независимое мнение об этом от меня.



8Оно не входит, насколько я знаю, ни в одну из уже построенных неклассических теорий.
9Выведется много бреда, поэтому будьте осторожны, но есть ссылка на книжку[link75]. Автор немного странноват, но, судя по его диссеру[link76], должен быть компетентен. Во всяком случае, публикации в признанных изданиях у него есть (PRE тот же). Конечно, он не мог удержаться от применения своей «информационной физики» к криптографии, поэтому получился параграф «Практическое использование разрушения информации: Криптография на основе детерминированного хаоса» (стр. 241). Вы ранее писали[link77], что это тупиковое и бесперспективное направление.
Гость (19/02/2014 12:24)   

В ту же копилку: Margolus-Levitin theorem[link79].
— unknown (19/02/2014 12:39, исправлен 19/02/2014 13:00)   

Нет. Ничего не гуглил по теме, честно :) Просто нравится придумывать концепции, даже не обладая достаточными почти никакими знаниями по предмету, а затем убеждаться, что всё уже придумано и на самом деле почти именно так и есть :)


Забавно, что некоторые воспринимают эту новоявленную экзотику слишком поспешно всерьёз и начинают накручивать на неё всякое фричество: то уже к торсионщине пытаются это прилепить, то к криптографии.


Надо заставлять себя заглядывать в будущее, но всему своё время, когда дело касается вопроса, что следует развивать и прорабатывать, а что нет.


P.S. Только сейчас прочитал в вики, что часть физиков критикует «теорию всего» за нарушение принципа бритвы Оккама.


Многие вещи в многомерных пространствах представить крайне трудно, но кое-что можно. Я когда в детстве пытался понять переход от декартовых координат к сферическим в n-мерном пространстве.

По поводу многомерных объектов и тяжёлого детства тоже вспомнил: как-то пытался представить четырёхмерный куб и вдруг внезапно представил. Решил зарисовать по логическому принципу, как соединение каждой вершины двух трёхмерных объектов. Спустя годы увидел, что именно так его проекцию и рисуют: чертёж один-в-один. Ни тогда, ни сейчас в таких случаях никогда не мог понять — это сам совсем с нуля придумал; просто где-то отложилось от бескосконечного просмотра похожей информации и собралось как мозаика; или может вообще видел уже где-то готовое краем глаза (слышал краем уха) — т.е. не сам, а просто вспомнил готовую информацию.

Гость (19/02/2014 13:59)   

Я думаю, фрики — это обычные люди с творческой жилкой, которые в своё время над собой не поработали. Когда понимаешь, что надо делать, по каким правилам и почему, осознаёшь всю трудность, творчество становится очень тяжелым. Но и отдача моральная от него есть, это тоже не последнее. Добавить что-то новое к знаниям человечества; сделать то, что ещё не делал никто; быть первопроходцем — это очень тяжело. Это погрязание в тоннах статей и литературы, месяцы (а то и годы) размышлений над темой. Многие такие вещи — это как труд всей жизни, т.е. «цель — довести до конца и опубликовать». И это принципиально не писательское творчество вида «что вижу, о том и пишу». Фрик над собой не работает, а творческая жилка у него хлещет, вот их и несёт.

Я, помнится, какие-то простые вещи тут хотел написать, стоят они в планах. Например — как объяснить спин из первопринципов. Вроде бы всё понятно, картинка в голове более-менее ясная, что из чего выводится, вроде вижу всю цепочку логических заключений между школьной физикой и основными понятиями о спине, но когда начал писать, увяз. Оказывается, этих звеньев в цепочке больше, чем я думал, и хотя сам я верю и когда-то усвоил это как данность, в чём-то убедился, что-то посчитал, но вот сейчас я уже не могу объяснить все звенья, каких-то логических стрелок не хватает. Т.е. я могу сказать, что они есть, но даже я сам их могу не вывести с ручкой и бумажкой сходу — надо будет сидеть, потеть, думать и разбираться. А рекомендовать другим дорожную карту «как прийти к этому знанию», если даже сам её представляешь лишь обрывочно — нехорошо. Пытался восполнить пробелы, полез в учебники и литературу. Узнал такие новые факты, о которых никогда ранее не слышал. Вся эта простая схема сведения получилась ещё более сложной и перегруженной (но и более интересной). Ну, и банально много всего. Вроде есть понимание, как кратко описать каждый шаг парой предложений, сохранив только смысл, но всё равно сложно это. Я думаю, что такая программа реализуема (в рамках одного поста объяснить то [на уровне смысла], на что у людей уходит много лет и не один месяц раздумий), но вот пока я её до ума так и не довёл. А тоже казалось, что просто. Т.е. даже то, что вроде бы «знаешь», попробуй ещё опиши, а что говорить про то, что заранее понимаешь, что не знаешь и близко?


На лурке есть статья[link80], она неплохая. По крайней мере, один раз её стоит прочитать. Я оттуда мало что нового узнал, но могу подтвердить, что текст соответствует тому, что я знал из обрывочных разговоров, рассказов и некоторых статей, и правильно отражает проблематику. Например, то, что

способов такого сведения существует по меньшей мере 10100

— недалеко от истины. ☺. Так что да, с бриттвой Оккама у струн не очень хорошо. С другой стороны, там есть ясный аргумент, почему в пространстве меньшей размерности создать теорию не получится.


Это разновидность «гиперкуба»[link81], называемая тессеракт[link82]. По ссылке есть видеоанимация его меняющихся проекций на двумерную плоскость или трёхмерное пространство. Выглядит заворащивающе, как будто это какая-то разновидность «невозможных фигур»[link83].


Криптомнезия[link84] — распространённое явление. ☺
Гость (19/02/2014 14:17)   

Строго говоря, там не совсем конструктивная критика. Есть вполне конкретный ответ: в первые моменты Большого Взрыва соотношение между силами (гравитационной и другими) было иным, и гравитацию нельзя было считать пренебрежимо малой. То же касается эффектов вблизи чёрных дыр. Т.е. когда-то была реальность (и, возможно, эта реальность до сих пор где-то во Вселенной существует), где наши теории заведомо не работают, поэтому попытка создать теорию вполне осмысленна. В других случаях энергии будет недостаточно, чтобы наблюдать эффекты квантовой гравитации.

К слову, есть ещё срач на тему тёмной материи[link85] и тёмной энергии[link86], которые существующей физикой, включая КТП, не объясняется никак. В частности, идея тёмной материи выросла из того факта, что в космических масштабах (вращение галактик), оказывается, не выполняются законы механики для движения тел вокруг центра тяжести. Т.е. мы знаем, что существующая КТП объясняет не всё даже из видимых явлений (хинт: космология), поэтому почему бы не попытаться разработать более общую теорию? Это не будет приумножением сущностей без необходимости.
— unknown (19/02/2014 14:17)   
Да, в вики уже смотрел. Именно так два варианта и зарисовал: «куб в кубе» и «куб рядом с кубом», соединённые дополнительными плоскостями. Третий вариант не представлял. Думаю, скорее всего, где-то всё-таки до того видел картинку в журнале, в телепередаче и т.д. Так что, криптомнезия, да.
Гость (19/02/2014 14:20)   

Фикс ссылки[link87].
— unknown (19/02/2014 14:21)   
Т.е. мы знаем, что существующая КТП объясняет не всё даже из видимых явлений (хинт: космология), поэтому почему бы не попытаться разработать более общую теорию? Это не будет приумножением сущностей без необходимости.

Тэги: [астрал], [телепатия]. Буквально на это и хотел указать: одно дело, когда всё хорошо описывается без лишних сущностей; другое дело — когда их явно не хватает для описания и все известные сущности вроде как перепробованы и исчерпаны.
Гость (20/02/2014 16:32)   

Есть знаменитое выражение про «неизмеримость квантовых состояний» (в ЛЛ-III это есть). Это не отменяет того, что |ψ〉 может быть восстановлено по ансамблю идентичных частиц. Я затрудняюсь дать точную трактовку этой «непосредственной неизмеримости», но смысл, кажется, там примерно такой:
Всё, что измеримо, по определению, является эрмитовым оператором. Физическую величину, которую с ним, оператором, ассоциируют, называют «(физической) наблюдаемой». В квантах используется множество операторов, и не все из них эрмитовы, поэтому не всякий оператор «соответствует наблюдаемой». В частности, с параметрами квантового состояния нельзя проассоциировать никакого измерения, т.е. эрмитового оператора, который бы соответствовал их измерению, поэтому квантовое состояние не является непосредственно измеримым (часто говорят просто «неизмеримым»).
Т.е. юмор в том, что даже в формальной терминологии, если использовать правильный квантовый язык, заложено то, что «их не пощупать». ☺

Кроме того, есть речи про «общую фазу» (это тоже классика, описана во всех учебниках). Смысл этих речей в том, что e|ψ〉 неотличимо от |ψ〉, если ничего другого нет (φ называется фазой). Если у вас есть ансамбль состояний только с одной фазой, вы не определите значение фазы, оно как бы не имеет никакого смысла и никак не отражается на измерениях. Оба типа состояний дадут одну и ту же плотность вероятности для всех наблюдаемых (например, координаты: |〈q|ψ〉|2). Вы также можете добавить одну и ту же фиксированную фазу вообще ко всем квантовым состояниям в мире (множитель e), и квантовая механика никак не изменится. Т.е. если мы вращаем всю квантовую систему относительно центра системы координат в векторном пространстве, у нас ничего не меняется, есть инвариантность.

Однако, если у вас есть два состояния, у которых разные фазы, разность фаз (но не сами их значения этих фаз!) можно узнать по ансамблю состояний. Грубо говоря, это соответствует тому, что у вас вектора состояний повёрнуты друг относительно друга. На физическом историческом языке иногда говорят, что состояния с разными фазами видны по их интерференции. Это дань исторической оптической аналогии: если вы описываете интерференцию волн в классической оптике, вы тоже работаете с фазами — теми же самыми множителями e. И когда говорят про интерференцию квантовых состояний, это иногда имеет буквальный смысл типа «электроны, проходящие через разные щели, получают разные фазы [фаза пропорциональна набегу (длине пути)], интерферируют друг с другом, и из-за этого мы на экране видим интерференционные полосы, как от волн». В общем случае, конечно, никаких волн нет (например, в спиновых пространствах), но состояния всё также «интерферируют», т.е. это выражение становится просто сленгом, отражающим лишь то, что у состояний есть фазы).

Иначе ещё можно сказать, что если у вас есть составная система, то подсистемы могут иметь разные фазы друг относительно друга, и каждое значение разности фаз соответствует своему состоянию совокупной системы. Общая фаза для подсистем (а, значит, и всего состояния) всё также не будет играть какой либо роли, её можно выбрать в расчётах, как удобно. Т.е. это сродни выбору конкретного базиса (системы отсчёта) при том, что поведение самой системы базисонезависимо.
Гость (20/02/2014 16:35)   
P.S. Т.е. эти факты про фазу тоже иногда относят на счёт «неизмеримости квантового состояния», потому что фазу вообще «не узнать», хотя разность фаз (а, значит, в некоторм смысле, и самое понятие фазы) важно для задания квантового состояния, как объекта.
Гость (20/02/2014 20:25)   
ну, теперь все понятно)) Понятно, что понятно не до конца, но последнии несколько постов "гостя" достаточно емко (в моем непонимании темы) и что главное сжато отразили передовую мысль человечества в области квантов, за что ему однозначно спасибо!
зы. (offtop) Единственное, что не совсем понятно, нафига все это (кванты, вселенная, объекты, субъекты)? Есть же какая-то цель у всего этого, к которой все это вместе и дружно движется так сказать)
— unknown (21/02/2014 11:54)   
Надо бы в раздел юмор, но новость[link89] слишком в контексте топика.
Гость (27/02/2014 15:20)   

На предмет цели, куда всё движется, есть разные мнения. Выбирайте вариант, который вам больше нравится:

п[link90]о направлению к Большому замерзанию, Большому разрыву, Большому сжатию или Большому отскоку

Ещё есть вариант

бесконечно повторяющейся циклической модели

Ссылка, кстати, хорошая, её стоит просмотреть. Это, своего, рода, общий знаменатель под тем, что науке ещё неизвестно.
Гость (23/03/2014 08:32)   

Направление мысли вполне прозрачное. Ещё где-то в детских книжках было: «пердставьте себе жучков-животных, живущих в двумерном пространстве; как вы думаете, смогут ли они определить, что их двумерное пространство искривлено?». Ответ, конечно, тривиален: есть множество методов, как это сделать. А раз так, возникает вопрос: можно ли повторить этот же трюк, но в трёхмерном пространстве? Точнее, не совсем так, но направление мысли то же: анализируя свойства пространства-времени, обнаружить, не вложено ли оно нетривиальным образом в какое-то ещё более широкое пространство со своими свойствами. Как жучки в двумерии пытаются определить, не находятся ли они, на самом деле, на сфере или на цилиндре, так и мы можем, в принципе, пытаться определить, не находимся ли мы все в большой чёрной дыре. ☺ Правда, мы не знаем, что должно быть в чёрной дыре, поэтому оценки довольно спекулятивны, но лучшей альтернативы пока нет.
Гость (09/05/2014 21:27)   

Owing to the form of Eq. (1), classifying the black hole quantum channel is reduced to the study of one-mode Gaussian (OMG) channels [12] // Стр. 2

Viewing the black hole in terms of the OMG channel construction will allow us to study how much information — encoded in a Gaussian state and sent into the black hole horizon — can be recovered by an outside stationary observer. // Стр. 2-3

Unlike the classical capacity, however, the quantum capacity of the OMG channels is (currently) known only in certain special cases [17, 18] and in general can only be estimated. This is unfortunate because even though at the time the black hole information problem was formulated the concepts of classical and quantum capacity of quantum channels did not even exist, today arguably more researchers are concerned with the fate of quantum information interacting with a black hole, which requires a calculation of the quantum channel capacity. // Стр. 3

Aside from characterizing the black hole OMG channel, we also extensively study the complementary quantum channel corresponding to the transmission of quantum information behind the horizon, to be decoded by a potential observer inside the black hole. This further elucidates the flow of quantum information in the black hole scenario. We find that the process of absorption of quantum states by the black hole horizon is, in the language of OMG channels, described by the family of conjugated channels. // Стр. 3

What are the consequences for (quantum) information retrieval from a black hole? // Стр. 14

OMG[link91]! Channels!

Типа Алиса вне чёрной дыры, а Боб внутри, за горизонтом. Между ними оптический канал. Анализируя его пропускную способность (capacity), якобы можно попытаться решить информационный парадокс об исчезновении информации. Народ анализирует классическую и квантовую пропускную способность, потом показывает, что квантовая пропускная способность (якобы самая интересная для этого случая) для каких-то параметров ненулевая, для других нулевая, а где-то просто неизвестна. Поскольку общая задача нахождения пропускной способности квантового канала не решена, анализируется модельное приближение — гауссовый канал. Для гауссового случая, впрочем, тоже не всё известно.
Гость (02/06/2014 07:00)   
В продолжение предыдущих постов: в Стекловке есть запись[link92] лекции Холево — введение в квантовую информатику для неспециалистов. Он один из тех, кто стоял у истоков этой области ещё в 70-ых. Конечно, какие-то математические знания для понимания требуются, но в целом это попыка элементарного введения, особенно вначале. Далее местами упоминаются более сложные вещи, которые в т.ч. и мне неизвестны (типа сходимости в слабой топологии), но на общее понимание доклада это не влияет. Круг освещаемых тем
  • Неравенства белла
  • ЭПР
  • Квантовые псевдотелепатические игры (игра Мермина-Переса)
  • Когерентные состояния (паршиво изложено, но на большее, видимо, времени было недостаточно; лучше бы было по классике: коммутационные соотношения, алгебры Ли и т.д.).
  • POVM
  • Церковь большого гильбертова пространства
  • Каналы, гауссовы каналы, и доказательство «minimum output entropy conjecture» (сглотнуто). Доказательство этой гипотезы — действительно один из серьёзных результатов последних лет:*
    Е[link36]динственный результат в нашей узкой области, наверное, который туда удалось бы пропихнуть — доказательство «Gaussian conjecture», про которое выше написал.
Если у кого-то есть свободный час времени, за который якобы можно понять, что есть квантовая теория информации, то можно посмотреть.

На том же youtube-канале есть[link93] лекция-введение в КТП. Я не смотрел всё, поглядел только начало. Объяснения очень индуктивные, детские, классические (как в книжках). В общем, всё очень понятно. Но как будет другим — не знаю. Вначале автор фактически один-в-один рассказывает квантование электромагнитного поля (решение задачи об осцилляторе — стандартная задача из книжек), просто полевыми операторами выступают не и (так называемые «лестничные» операторы или операторы рождения и уничтожения), а произвольные полевые — ψ̂ и ψ̂. В общем, «вторичное квантование» в самой его типичной реинкарнации.

Интересно, что по криптографии там тоже[link94] доклад есть. Докладчик — некто Зубков. Я ожидал, честно говоря, худшего, но оказалось, что не всё так плохо. Первая половина доклада про исторические шифры и военное крипто. Во второй он что-то пытался рассказать про современные вещи, но вообще не вдаваясь в детали. На 45:25 он говорит про (см. слайды) «условия максимальности периода». Никогда о таком не слышал. Про ГПСЧ он говорит, что это, с точки зрения математики, конечный автомат, так что как ни крути, случайности там не будет. Якобы его можно представлять через p-адические числа и ещё так анализировать(?). Про то, что настоящая случайность берётся только из физических процессов, нет ни слова, как и про то, что современная криптография в плане математики теснее всего связана с теорией сложности (complexity theory).


*Оригинальное доказательство тут[link95]. На неделю позже вывели[link96] в архив статью с альтернативным и более простым доказательством (на основе теории мажоризации), где доказано более сильное утверждение, из которого следует исходное. Статья прошла в Nature Communications. Недавно вышла ещё одна[link97] с обсуждениями результатов и их влиянием на разные вопросы в области теории квантовых коммуникаций. По формату PRL, куда её послали — не знаю. Кроме того, обещают ещё одно альтернативное доказательство на основе методов КТП.

Вообще, гауссова гипотеза (дословно — «гипотеза об оптимальных гауссовых ансамблях») — древняя штука. В случае обычных (тепловых) состояний она сводится к другой гипотезе — о том, что энтропия на выходе канала минимизируется, если на вход подан вакуум. Последние лет 15 эту последнюю гипотезу пытались доказать, и всё было безуспешно. В каких-то тривиальных случаях она доказывалась элементарно, в общем случае — нет. Ну, а исходня гипотеза, в самом общем её виде, открыта до сих пор. Например, все наши расчёты опираются на верность этой гипотезы. Если гипотеза будет неверна, значение результаты полностью не потеряют, но их фундаментальность поуменьшится.

Смысл для людей со стороны можно пояснить примерно так: у вас есть электромагнитные волны, и вы передаёте через них информацию. Сколько бит можно передать (даже в случае простейшего теплового шума)? Этот же вопрос можно поставить про оптоволокно, модель которого, как и модель free space transmission, примерно описывается одним из типов гауссовых каналов. Плюс гауссов шум — он самый поганый для передачи информации, поэтому даже если канал где-то не такой, это даёт оценку по наихудшему случаю.

Если вы ничего не знаете про квантовую механику и попытаетесь посчитать пропускную способность такого канала, вы получите абсурдный, в общем случае, результат: C=log2(1+P/N), где P — мощность сигнала, а N — шум. Это стандартная формула из учебников по теории информации. Итак, если шум стремится к нулю, пропускная спсобность стремится к бесконечности, что физически абсурдно. Если вы остаётесь в рамках классической теории, у вас нет возможности получить правильный результат. Настоящая теория информации должна быть квантовой. В последней как раз оказывается, что N≠0, потому что есть неуменьшаемый вакуумный шум (та самая знаменитая нулевая энергия). После его учёта получается вполне корректная формула. Результат в некотором смысле фундаментален, потому что человечество уже больше 100 лет передаёт информацию через электромагнитные волны, а пропускную способность такого канала знало только приближённо (тут надо произнести ряд оговорок про модель, но даже простая трактовка не сильно искажает суть).

Доказательство, как и положено, не обошлось без политики, срачей, выяснения и разрывов отношений и вообще. Когда на кону стоит, чьи фамилии и в каком порядке будут написаны в основополагающем результате, в ход идёт всё. Даже самый надёжный гарант может кинуть. Никому нельзя доверять, даже своим. Не стоит делиться наработками, частичными результатами, обсуждать их. Даже самая мелкая утечка может повлиять на то, что конкуренты получат доказательство немного раньше, а вас пошлют лесом. Большие деньги — это большая грязь. Вот в большой науке то ж самое. ☺

К слову, я об этом доказательстве узнал из архива, хотя его во многом сделали у нас люди, которых вижу ежедневно. Конечно, я знал, что они работают над этой проблемой, но не знал, что уже так близко подошли к решению, что вот-вот уже статья будет. Когда статья уже лежала в архиве (я и большинство других об этом ещё не знали), нам о существовании доказательства шёпотом сообщали окольными путями третьи люди под NDA. Это такой цирк и маразм, что даже вообразить сложно.
— unknown (02/06/2014 11:11, исправлен 02/06/2014 11:12)   

Где-то видел запись драки между докладчиками на научной конференции, примерно как это выглядит иногда в парламентах.


Для таких ваших постов нужен даже не ФПП[link98], а какой-то другой ФП. Потому что не всегда можно осмыслить инфу сразу, со всеми ссылками, а затем спустя чуть ли не годы может внезапно понадобиться. И приходиться мучительно подбирать ключевые слова для поиска.

Гость (02/06/2014 13:49)   

Classical noise assists the flow of quantum energy by `momentum rejuvenation'[link99]
(Submitted on 30 May 2014)
Гость (03/06/2014 09:17)   
Еще из трогательных новостей: квантовый компьютер опять запретили[link100].
Гость (17/01/2015 21:47)   

Аннотация доклада на тему «рисования стрелок» [1][link101] и сверхплотного кодирования [2][link102], [3][link103] [слив без ссылки]:

The communication power of quantum theory

[censored] We first address the question: How much information can a transmitted quantum system fundamentally communicate? We briefly introduce the principle of quantum information causality, which states the maximum amount of quantum information that a transmitted quantum system can communicate as a function of its Hilbert space dimension, independently of any quantum physical resources previously shared by the communicating parties.

The second part of the talk considers superdense coding within the framework of general probabilistic theories and addresses the question of why in quantum theory, no more than two bits can be communicated by transmission of a single, entangled, qubit. We introduce hyper dense coding in general probabilistic theories: superdense coding in which N>2 bits are communicated by transmission of a system that locally encodes at most one bit, and present protocols with N arbitrarily large. Our hyperdense coding protocols imply super additive classical capacities: two entangled systems can encode N>2 bits, even though each system locally encodes at most one bit. Our protocols violate either a reversibility condition or tomographic locality.

Среди работ автора[link104] пока нашлось только это[link105] по теме (PRL-2013). Ещё обнаружилась диссертация автора[link106], там этому посвящена четвёртая глава. По сути это попытка вывести самые общие ограничения на передачу информации опираясь (только?) на принцип причинности.

Бакалавр и диссер были в Кембридже, руководитель — тот самый Кент[link107], который изобрёл релятивистский BC. А после диссера он, видимо, перебрался[link108] к Массару[link109]/Пиронио, т.к. тот, как и автор, тоже занимается много неравенствами Белла.
Гость (18/01/2015 03:59)   
В тему квантовой термодинамики[link111]:

F[link112]urther, we quantify the time or number of steps it takes to cool a system to any particular temperature. Our argument relies on the heat capacity of the bath being positive for large temperatures, and we show that if this is not the case then perfect cooling in finite time is in principle possible. Our results also clarify the connection between two versions of the third law (the Unattainability Principle, and the Heat Theorem), and place ultimate bounds on the speed at which information can be erased.

Т.е. на скорость уничтожения информации тоже могут быть наложены какие-то фундаментальные физические ограничения.
Гость (11/02/2015 09:48)   
Немножно на тему бомовской[link40] механики и свободы выбора[link113]:

Is Bohmian Mechanics incompatible with free will/choice?[link114]

Из привата на эту тему: доклад

Is Bohmian Mechanics self-consistent?

Abstract: According to quantum theory, the outcomes of measurements are generally not deterministic. "Single-world" theories (such as Bohmian mechanics) add additional elements to quantum theory in order to restore determinism. In this talk, I will argue that such single-world theories cannot be self-consistent in the following sense: any attempt to use a single-world theory to describe an observer who himself applies the theory necessarily results in a contradiction. © Renner

Теория просто обязана быть внешней по отношению к наблюдателю? Включение наблюдателя в часть эксперимента в принципе невозможно? Проблема измерений[link115] — лишь проблема классичности нашего сознания, а потому принципиально неразрешима?
Гость (11/02/2015 10:03)   

Ещё один[link116] тематический канал. Он от IHES, и там много на французском.
— unknown (11/02/2015 10:42)   

Может есть какие-то неизвестные уровни реальности. И если представить, что их все можно учесть, то и наблюдателя нет, времени нет, измерений нет. Есть только супердетерминизм, заданный некими высшими уровнями абстракций. Т.е., эдакий солипсизм наизнанку: не наблюдатель есть — мира нет, а мир есть — наблюдателя нет. Поэтому и все теории неполны.
Гость (11/02/2015 10:56)   

Хорошо сказано. ☺
— unknown (11/02/2015 10:59)   
В процессе познания мира выяснилось: мир объективно есть, нас не существует. ☺
Гость (11/02/2015 13:01)   
Только «не существует» ровно в том смысле, что не существует по отдельности (мы — отдельно, мир — отдельно), т.е. мы являемся частью мира и не можем вынести себя за его рамки.
— unknown (11/02/2015 13:42)   

И в этом смысле тоже. Хотя, кому-то хочется переселиться на другой глобус, кому-то — вынести себя в иные миры (многомировая интерпретация, видимо, навеяна чем-то таким). Это уже вопрос философских предпочтений.
Гость (24/02/2015 20:42)   
— Всего существует четыре фундаментальных взаимодействия между частицами: (1) Гравитация, которая подчиняется данному закону обратных квадратов: [...]
— Окей.
— (2) Электромагнетизм, который подчиняется данному закону обратных квадратов: [...] а также уравнениям Максвелла.
— А также что?
— (3) Сильное ядерное взаимодействие, которое подчиняется... эээ... ну, эээ... оно удерживает протоны и нейтроны вместе.
— Понятно.
— Оно сильное. И (4) Слабое взаимодействие. Оно [бу–бу–бу] радиоактивный распад [бу–бу–бу].
— Это не предложение вообще. Вы лишь сказали "радио–
— И вот они все четыре фундаментальных взаимодействия! Из этих четырёх взаимодействий есть одно, которое мы до конца не понимаем.
— Это слабое или сильное?
— Это гравитация.

Очень тонкий юмор[link117]. :)
— unknown (24/02/2015 21:18)   
Ждём подтверждения удержания вместе гравитонов и гравитино[link118]:

Ряд физиков отвергает саму гипотезу о гравитоне как несостоятельную[6]. Например, если гравитоны существуют, то они должны излучаться чёрными дырами, что, вероятно, противоречит Общей теории относительности.[источник не указан 952 дня]
Гость (26/02/2015 00:31)   


ровно на 01:29:55 "Причины о которых я не могу здесь распространяться" что это значит? наука имеет гриф секретности?
Ведь после этой фразы (допускающей пробел в доказательствах) вся почти двухчасовая лекция на смарку!
Гость (26/02/2015 02:20)   

Ни на 0:29:55, ни на 01:29:55, ни на 01:39:55 нет слов «Причины о которых я не могу здесь распространяться». Что-то вы напутали.
Гость (26/02/2015 03:51)   

Это было в лекции Холево[link92], предложенное Гостем 02/06/2014 07:00[link119]
Гость (28/02/2015 00:34)   
Так бы сразу и сказали.


Это всё, что вы поняли из этой лекции? Есть вещи, о которых простым смертным знать не положено. Любая лекция имеет строгое ограничение по времени, в него надо уложиться, это особое и сложное искусство. В данном случае заранее понятно, что что-то пришлось оставить за кадром просто из-за недостатка времени.


Гуманитарное мышление? Ага, вот так последовательно рассказывал полтора часа, выводя одно из другого, а потом тонкость одного определения не обосновал, и сразу всё равнее сказанное пошло «насмарку». Если уж придираться к мелочам, то из-за необоснованной оговорки пойти насмарку может (в принципе) только то, что на неё завязано и сказано после неё, а не всё, что сказано до неё.


Значит то, что ему некогда вдаваться в такие тонкости:

В[link120]полне положительность требуют, что при работе с запутанными состояниями не получалось дичи (иначе они органично в общую замкнутую консистентную схему не впишутся). Тем не менее, есть некоторые работы и по изучению CP-отображений, не являющихся вполне положительными. Ещё можно показать, что вполне положительность — более сильное требование, т.е. просто положительность из неё уже следует.

Об этом же в других источниках:

T[link121]he matrix transpose operation ρ → ρt is a prominent example of such an unphysical operation.

T[link122]his is false when the system is entangled with something else. The classic example is the transpose on the system of interest.

S[link123]haji and Sudarshan argued in a Physics Letters A paper that, upon close examination, complete positivity is not a requirement for a good representation of open quantum evolution. Their calculations show that, when starting with some fixed initial correlations between the observed system and the environment, the map restricted to the system itself is not necessarily even positive. However, it is not positive only for those states that do not satisfy the assumption about the form of initial correlations. Thus, they show that to get a full understanding of quantum evolution, non completely-positive maps should be considered as well.

Если не усилить положительность вполне положительностью, то канал при действии на открытые системы (например, часть запутанной пары) мог бы переводить некоторые физические состояния в нефизические, что запрещено законами. Чтобы определение лучше соответствовало физике возможного, оттуда убрали такие патологические случаи пофиксом определения.
Гость (16/03/2015 15:13)   

Наоборот, это гуманитарное мышление предполагает "выводы между строк" и "само-собой разумеющееся" из чего-то там... А наука дело точное! А по Вашей логике так можно и безопасность строить оставив где-то "небольшую дырочку" (через которую все и полезет), а остальное будет "последовательно полтора часа, выводя одно из другого".
— unknown (16/03/2015 15:55, исправлен 16/03/2015 16:00)   

Это только в тех областях, где всё уже изучено и понятно много веков как.



Это редукционистский подход. Хотя редукционизм иногда вполне обосновано критикуется[link124], как на теоретическом уровне, так и за неумелое использование, но ничего особенно лучше для доказуемой безопасности не придумали. См. цитаты в соотв. коменте[link125].


Да и «наука — дело точное» только в школьных учебниках, а внутри там такой же редукционизм, конвенционализм и пр.


Как говориться, "доказательство только тогда становится доказательством, когда совершён некий социальный акт 'принятия его как доказательства'"[link126].

Гость (16/03/2015 19:00)   

Из какой именно цитаты был сделан такой странный вывод?
— pgprubot (31/07/2015 00:05)   
T[link127]he ultimate physical limits of privacy
<...>
The questions, and my answers, are below.
<...>
  1. Does the universe provide us with a minimum level of information security?
  2. For example, is my information safe from entities outside the light-cone I project?
  3. Assume that there are distant alien cultures with infinite life spans – would they always be able to wait long enough for my light cone to spread to them, and then have a chance of detecting my “private” information?
  4. Does living in a black hole provide privacy? Couldn’t they follow you into the hole?

Из ответа на последний вопрос:

No, I would not recommend jumping into a black hole as a way to ensure your privacy. For one thing, you won’t get to enjoy the privacy for long (a couple hours, maybe, for a supermassive black hole at the center of a galaxy?) before getting spaghettified on your way to the singularity. For another, as you correctly pointed out, other people could still snoop on you by jumping into the black hole themselves—although they’d have to want badly enough to learn your secrets that they wouldn’t mind dying themselves along with you, and also not being able to share whatever they learned with anyone outside the hole.

© Ааронсон.
— ОляВедьма (03/08/2015 14:59)   
Знаю что нарвусь на удаление, но я вполне серьезно! Без стеба.
Есть "эзотерическая" организация (может и не одна, называть не буду чтобы не было рекламы) которые распространяют цифровые материалы "для внутреннего пользования" и там есть везде предупреждение типа: "копирование защищено психотронной защитой, нарушение повредит вашему здоровью"... Смешно? Тысячи адептов ВЕРЯТ в это и честно говоря у меня у самой не было желания копировать все это. У меня деловое предложение! Я даю такой "защищенный материал" смельчаку который свободно это копирует (ну там другу даст или по дискам копирует) и смотрим что будет.
PS Это было бы "смешно", если бы тысячи людей не верили (((
— ОляВедьма (03/08/2015 15:03)   
Да не подумайте что это скрытая реклама какой-то секты. Я дам записи где нет их координат а ваша задача это просто скопировать куда-то. Не распространять можете просто болванки накатать или знакомому дать "для эксперимента" и все!
— ОляВедьма (03/08/2015 15:07)   
Если интересно для технарей то в записях шумы, типа "для настройки сознания на нужные режимы для медитаций". То есть "проповедей" там нет, там голос только о "защите" предупреждает. Мне главное эксперимент (копировать несколько раз и смотреть на свое здоровье) Если будет откровенное опровержение специалиста (копировал до дури и здоров), то клево, а если другие косяки ("да мне лень", "да неинтересно" и тем более если кто-то скопировав заболеет или умрет) то грустно(((
— cypherpunks (12/12/2015 05:03)   
Мне кажется в тему: Какие есть химическо-аутотреннинговые средства для повышения самоконтроля на подсознательном уровне? (чтоб по пьяне не проболтался, чтоб во сне не разговаривал) Вопрос далеко не шуточный. Пока кроме "не пить" и "пить снотворное" (для сна без болтовни) мыслей нет(
— cypherpunks (01/01/2016 11:53)   
Прошу прощения, что сюда, т.к. тему "Пси-воздействия вместо криптоанализа" закрыли. Это не стеб, может быть одно из нескольких сообщений по существу.

Помните, смеялись над Половко? Так вот, я еще форумы почитал много всякого и... забыл. А сейчас перед Новым Годом были ответственные дела, но я никак не мог собраться, потому что мой нервяк (как обычно перед командировками, крупными сделками) усилился, а как Новый Год прошел – тут эйфория настала. Это впринципе ни о чем, но натолкнуло меня на размышления и картинка более сложилась. Я вспомнил точ-в-точ описание человека на эту тему, который писал "сказку" о психотронных установках, которые используют власти для сдерживания толпы и как их перед праздниками врубают в одну сторону, а во время праздника в "эйфорию". Сложил это с байками Невзорова в его серии "против РПЦ", где он рассказывает про дрессировку толпы постами, разговениями и прочим. Вспомнил то, что писала Половко (и другие деятели) о том, как изолироваться от ЭИ психотронного воздействия.
И подумал о том, что можно рассуждать про "неудачные опыты до сих пор ведутся", вспоминать как "американцы в Грузиях и Африке" со своими СВЧ установками "обделались", про "систему ХАРП которая только-только запущена", но что если еще давным-давно, лет 50 назад еще при совке ученые нашли тупой способ воздействия на конкретные центры нервной системы человека (по теории такой же тупой как звук частотой 3 Гц), а далее подобрали такую частоту или модуляцию, чтобы она легко проходила через разные естественные экраны (растительность, дома, одежду) и используют это либо на телебашне, либо на каких-то административных антеннах. Может быть используются даже арматура зданий и другие металлические коммуникации и элементы строений. При этом себестоимость использования (не разработки, а изготовления простого модулятора+передатчика и потребление) такое низкое, что данные установки могут работать и в "кризисы", и в "перестройку"... И возможно, как общее фоновое воздействие (чтобы у народа не было желания совершить переворот), так и частное (во время праздников, саммитов, выборов).
Если в свое время запустили единую систему контроля за ядерными ракетами (низкочастотное радиоизлучение, покрывающее кажется северную часть земного шара которое в случае прекращения на 4 часа провоцирует запуск ракет по мишеням "по умолчанию" излучатели у США на Аляске, в СССР построены на Кольском п-ве), то почему не сделать тоже самое для контроля сотен миллионов людей во имя госбезопасности?
Ну и конечно вопрос, как от этого защититься? Дома в сетке Фарадея, одежда из экранирующих материалов и все?
И что меня волновало всегда, когда я изучал вопрос государственной психотроники. Как экранируют самих лидеров держав и их функционеров (в т.ч. операторов психотронных центров)? Ясно, что чиновники и политики могут быть марионетками, но ведь далеко не все, а тем более те кто выполняют роль "пастырей для овец" (руководители спецслужб, военные занимающие ключевые посты) должны быть защищены!
Прошу еще раз прощения за возможно не совсем по теме форума, но мой вопрос конструктивный и если ответы не будут "троллится", в них не будет тупых шуток и стеба, то тема не испортится!
Я надеюсь услышать конструктив. Не байки Половко о грязи на бачках, не про "всемирный массонский заговор", а подкрепленные логикой и знаниями физики и биологии ответы хотя бы на перечисленные мной вопросы.
Спасибо и всех с Новым Годом!
— pgprubot (02/01/2016 17:05, исправлен 02/01/2016 17:06)   

В старых ветках эти вопросы действительно обсуждались, и вряд ли к этому можно добавить что-либо нового. Если все ранее произнесённые аргументы лично вы считаете неубедительными и продолжаете верить в то, что хочется — это ваше личное дело.


Неявно можно действовать либо очень сильным СВЧ, либо инфразвуком. Собственно на психику действует только инфразвук, да и то не очень управляемым образом. Излучатели инфразвука — не иголки в стоге cена, их массовое существование спрятать бы не удалось, об этом бы все знали.


На тему сомнительных советских программ (а также массы других малоизвестных фактов), что это было, зачем и почему, есть хорошее интервью с Александровым в двух частях [1][link128], [2][link129].

— cypherpunks (06/01/2016 17:50)   
Уважаемый, не ведите себя так, будто вы на эзотерическом форуме или на зарплате у РАН.
Я не "Ольга Ведьма" которая тут тусовалась и если вы перестанете вести себя как неудавшийся ученый в секте, пытающийся что-то доказать мошенникам и их полуграмотным адептам и перечитаете сообщение выше, то поймете что вопросы были таковы:
Возможно ли излучение таким же способом (буквально одной антенной все северное полушарие) каким контролируют подлодки и ракетные шахты низкочастотным ЭИ?
Заметьте, ни слова об "инфразвуке", а если уж говорить о нем, то что мешает построить подземную установку одну на каждый город? (у меня есть устойчивая версия, что людям "выезжающим загород" становится легче не потому что "природа, прички поют и нет этих городских")
Возможно ли использование других коммуникаций для проведения данного излучения? (водопровод, газопровод, арматура, любые электрические сети: телефон, кабельное тв, электричество)
Ну и в наказание вам за то что вы стали беситься, разговаривая с нами как с лохами, которые носят деньги "колдунам", я дополню версии воздействия современными особенностями, невозможными 20-30 лет назад – через базовые станции сотовой связи либо дополнительными модуляциями, либо может быть эти способы СВЧ воздействий на живые объекты близки к стандартам используемом в сотовой связи.
А на ваш контраргумент: "очень сильным СВЧ", отвечу, что слышал от специалистов (не "магов" еще раз вам говорю, а например в физиотерапии), что для воздействия на живой объект решающим является не мощность излучения, а частоты, модуляции.
В то же время я крайне не верю в способ, предложенный Половко (закутаться в фольгу), здесь экран поумнее нужен, а то и активную защиту (отвечать на излучение в противофазе и т.п.)
— pgprubot (11/01/2016 17:50)   

Есть много каких хорошо проникающих излучений, только про способы чудодейственного контроля мозгов с помощью этих излучений науке ничего неизвестно.


Невозможность сохранить информацию про этот проект втайне. Как показывает история, даже информация про единичные (на всю страну) засекреченные подземные проекты быстро становится секретом Полишинеля.


Дело не просто в воздействии в плане вреда/пользы для здоровья, а в прямом воздействии на психику, что совсем другое. Пока вы не укажете на научно установленные факты о существовании таких воздействий, говорить не о чем. Способов прямого воздействия на психику не так много — я знаю только про инфразвук и психоактивные (галлюциногенные, психоделические) вещества.


Есть в этом доля правды, но человек, в общем, низкочастотное и немагнитное (неметаллическое) существо, поэтому способов воздействия на его психику мало. Скорей всего, остаются только звуковые волны. Кстати, даже в акустических пушках[link130] всё, как вижу, сводится только к мощности, а не к модуляциям, и при инфразвук там ни слова.
— cypherpunks (18/01/2016 02:23)   


А здесь не нужно контролировать мозг как Мелофон с Алисой. Достаточно просто включать "ступор" на уровне нервной системы, чтобы человек боялся (ну как страх работает в курсе я думаю)



А вы разве не помните как возмущался ваш Кругляков (тот что в команде Александрова) про всякие секретные "антинаучные" лаборатории?



Легкий страх от инфразвука испытываешь иногда даже(!) когда под окнами припаркован автомобиль со включенным двигателем.
Тоже самое касается и других видов излучений (мерцание света, которые при грамотно-подобранной частоте вредят не только эпилептикам, ультразвуковые колебания, отрицаемые вами ЭИ) все это известно в области физиотерапии[link131].



Не было бы шокеров, если бы было так, как вы пишете!
Нервная система это кучи сигнальных (электрических) магистралей. Особенно в районе спинного мозга, головы и сердца. Об этом хорошо известно врачам! Есть множество способов воздействия на организм именно в через ЭИ, а не звуком!

Неужели по вашему никто не мог чисто экспериментальным (даже без специализированной научной базы) путем подобрать частоты-модуляции, которые вызывали бы у людей нужные реакции, дальше подобрать сигналы так, чтобы искажения которые накладывают стены и городские коммуникации, в том числе городские помехи не просаживали и не искажали бы полезную составляющую сигнала? (чтобы не лохануться как Гитлер распространивший в Великобритании пластинки с инфразвуком и не учел, что домашние грамофоны не воспроизводят нужные для воздействия частоты)

PS Вернемся к практике. Лично мне как чувствительному человеку легче в ванной, туалете, в некоторых(!) подвальных помещениях, при полете на самолете (а там, кстати, ваш инфразвук от двигателей ой-ей-ей) и иногда чуточку легче под двумя-тремя толстыми одеялами или в завешанном зимней одеждой платяном шкафу (на последние мои аргументы прибегут фрейдовцы которые начнут меня лечить про "ощущение безопасности в норке пришедшее с внутриутробного детства"?)
— pgprubot (20/01/2016 06:47, исправлен 20/01/2016 06:47)   

Нет, не помню. Наш — это чей? Вы обращаетесь не пойми к кому.


Чувствую, вы в упор не видите разницу между скрытием одной закрытой лаборатории на всю страну и скрытием тысяч огромных подземных облучателей, установленных в каждом городе.



Вот только пока никто не придумал, как из этого сделать оружие массового покорения. У людей бывает паранояльный шизофренический бред. Опровергнуть его логическими доводами невозможно. В реальности мне неоднократно доводилось сталкиваться с шизофрениками, параноиками (в медицинском смысле) и слушать их рассказы. То, что вы здесь пишете, от того, что говорили они, по логической состоятельности отличается мало.



Т.е. вы не понимаете, что такое магнитный материал? Отличием электропроводности человеческого тела от металлов не интересовались? Про диэлектрики и пробой диэлектриков никогда не слышали?



Вам открыть тайну о том, что диэлектрики тоже состоят из заряжённых частиц, как-то худо-бедно проводят ток, да ещё и электризуются иногда, причём ничего из перечисленного не делает их магнетиками[link132]?



Есть. Как раз можете поинтересоваться, какой величины ЭИ для этого требуется и почему, чтобы понять, почему действие через ЭИ настолько малоэффективно.



Обычно доказывается знание, а не незнание, и тяжесть доказательства всегда на том, кто высказывает тезис, а не опровергает его. Вы высказали, что можно, потому что не доказано обратное, но никаких веских аргументов в пользу существования я от вас не услышал.



Чем бы ни было это вызвано, оно не имеет никакого отношения к тому тезису, который вы высказываете про всепроникающие излучения для массового контроля сознания.



Нет, вы, наверно, просто охлофоб[link133] (антипод клаустрофоба).


Ссылки
[link1] https://help.ubuntu.com/community/SecureDeletion

[link2] https://ru.wikipedia.org/wiki/Квантовое_бессмертие

[link3] https://ru.wikipedia.org/wiki/Аннигиляция

[link4] https://en.wikipedia.org/wiki/POVM

[link5] https://en.wikipedia.org/wiki/Information

[link6] http://www.pgpru.com/comment48305

[link7] https://en.wikipedia.org/wiki/Interpretations_of_quantum_mechanics

[link8] https://ru.wikipedia.org/wiki/Квантовая_гравитация

[link9] http://www.nkj.ru/archive/articles/10957/

[link10] http://www.pgpru.com/comment58705

[link11] http://benasque.org/2014QIP/talks_contr/031_preskill-QIP2014.pdf

[link12] https://en.wikipedia.org/wiki/John_Preskill

[link13] http://www.pgpru.com/comment66069

[link14] http://www.pgpru.com/comment52788

[link15] http://www.iqi.caltech.edu/people.html

[link16] https://www.pgpru.com/comment76735/show?time=2014-02-14%2000:29:04

[link17] http://olegart.ru/wordpress/2013/12/23/4062/

[link18] http://www.pgpru.com/comment69270

[link19] http://www.pgpru.com/comment69402

[link20] http://www.pgpru.com/comment69435

[link21] http://www.pgpru.com/comment69444

[link22] http://www.pgpru.com/comment69458

[link23] http://www.pgpru.com/comment69621

[link24] https://en.wikipedia.org/wiki/Quantum_Zeno_effect

[link25] http://www.pgpru.com/comment69623

[link26] https://en.wikipedia.org/wiki/Qutrit

[link27] https://en.wikipedia.org/wiki/Ternary_computer

[link28] http://www.pgpru.com/comment56608

[link29] http://www.pgpru.com/comment62261

[link30] http://scholar.google.de/citations?user=fe9uXzkAAAAJ&hl=en

[link31] http://www.pgpru.com/comment76715

[link32] https://ru.wikipedia.org/wiki/Калибровочная_инвариантность

[link33] https://ru.wikipedia.org/wiki/Термодинамические_потенциалы

[link34] https://ru.wikipedia.org/wiki/Распределение_Гиббса

[link35] https://ru.wikipedia.org/wiki/Эргодическая_гипотеза

[link36] http://www.pgpru.com/forum/kriptografija/voprosypokvantovymkanalamihmodelirovanieiprimenenie

[link37] http://www.pgpru.com/comment48758

[link38] http://www.pgpru.com/comment76695

[link39] http://www.pgpru.com/comment69514

[link40] http://www.pgpru.com/comment69471

[link41] https://en.wikipedia.org/wiki/Noether's_theorem

[link42] https://en.wikipedia.org/wiki/File:Standard_Model_of_Elementary_Particles.svg

[link43] https://ru.wikipedia.org/wiki/Принцип_наименьшего_действия

[link44] https://en.wikipedia.org/wiki/Functional_integration

[link45] http://www.pgpru.com/comment56705

[link46] http://www.pgpru.com/comment76831

[link47] http://www.pgpru.com/comment76838

[link48] https://en.wikipedia.org/wiki/Measurement_problem

[link49] https://en.wikipedia.org/wiki/Quantum_decoherence

[link50] https://en.wikipedia.org/wiki/Wigner-Weyl_transform

[link51] http://www.univie.ac.at/nuhag-php/bibtex/open_files/10541_M.deGosson_F.Luef-BornJordanQuant.pdf

[link52] http://www.pgpru.com/comment52117

[link53] http://www.pgpru.com/comment60675

[link54] http://www.pgpru.com/comment52116

[link55] http://www.pgpru.com/comment73684

[link56] http://www.pgpru.com/comment73298

[link57] http://www.pgpru.com/comment76671

[link58] https://en.wikipedia.org/wiki/Bekenstein_bound

[link59] https://en.wikipedia.org/wiki/Bremermann's_limit

[link60] http://bnw.im/p/DR0GNQ

[link61] https://en.wikipedia.org/wiki/Digital_physics

[link62] https://ru.wikipedia.org/wiki/Цифровая_физика

[link63] https://en.wikipedia.org/wiki/Digital_philosophy

[link64] https://ru.wikipedia.org/wiki/Цифровая_философия

[link65] https://en.wikipedia.org/wiki/Digital_probabilistic_physics

[link66] https://ru.wikipedia.org/wiki/Тепловая_смерть_Вселенной

[link67] https://ru.wikipedia.org/wiki/Дирак,_Поль_Адриен_Морис

[link68] https://en.wikipedia.org/wiki/Spherical_harmonics

[link69] https://ru.wikipedia.org/wiki/Квантовая_химия

[link70] https://ru.wikipedia.org/wiki/Корректно_поставленная_задача

[link71] https://ru.wikipedia.org/wiki/Теория_всего

[link72] https://ru.wikipedia.org/wiki/Фундаментальные_взаимодействия

[link73] https://ru.wikipedia.org/wiki/М-теория

[link74] https://ru.wikipedia.org/wiki/Петлевая_квантовая_гравитация

[link75] http://www.proklondike.com/var/file/vstovskiy_elements_math_physics.zip

[link76] http://www.dissercat.com/content/fraktalnaya-parametrizatsiya-struktur-v-metallakh-i-splavakh

[link77] http://www.pgpru.com/comment19079

[link78] http://www.pgpru.com/comment76870

[link79] https://en.wikipedia.org/wiki/Margolus-Levitin_theorem

[link80] https://lurkmore.to/Теория_струн

[link81] https://ru.wikipedia.org/wiki/Гиперкуб

[link82] https://ru.wikipedia.org/wiki/Тессеракт

[link83] https://ru.wikipedia.org/wiki/Невозможная_фигура

[link84] https://ru.wikipedia.org/wiki/Криптомнезия

[link85] https://ru.wikipedia.org/wiki/Тёмная_материя

[link86] https://ru.wikipedia.org/wiki/Тёмной_энергии

[link87] https://ru.wikipedia.org/wiki/Тёмная_энергия

[link88] http://www.pgpru.com/comment76851

[link89] http://www.rosbalt.ru/style/2014/02/21/1236052.html

[link90] https://ru.wikipedia.org/wiki/Нерешённые_проблемы_современной_физики

[link91] http://arxiv.org/abs/1405.1097

[link92] https://www.youtube.com/watch?v=YntOY0Fkz9s

[link93] https://www.youtube.com/watch?v=7K3-SmqcHAI

[link94] https://www.youtube.com/watch?v=U-6ZRgIBeQQ

[link95] http://arxiv.org/abs/1312.2251

[link96] http://arxiv.org/abs/1312.3545

[link97] http://arxiv.org/abs/1312.6225

[link98] http://www.pgpru.com/biblioteka/osnovy/fondpoleznyhpostov

[link99] http://arxiv.org/abs/1405.7914

[link100] http://phys.org/news/2014-05-analysis-potential-quantum.html

[link101] http://www.pgpru.com/comment76833

[link102] http://www.pgpru.com/comment56599

[link103] http://www.pgpru.com/comment57321

[link104] http://arxiv.org/find/quant-ph/1/au:%20Pitalua_Garcia_D/0/1/0/all/0/1

[link105] http://arxiv.org/abs/1210.1943

[link106] https://www.repository.cam.ac.uk/handle/1810/245205

[link107] https://www.pgpru.com/comment73659

[link108] http://www.dpitaluagarcia.org/

[link109] http://www.pgpru.com/comment40651

[link110] http://fr.arxiv.org/abs/1412.8483

[link111] http://www.pgpru.com/comment76672

[link112] http://fr.arxiv.org/abs/1412.3828

[link113] http://www.pgpru.com/comment74189

[link114] https://www.physicsforums.com/threads/is-bohmian-mechanics-incompatible-with-free-will-choice.674784/

[link115] http://www.pgpru.com/comment76863

[link116] https://www.youtube.com/channel/UC4R1IsRVKs_qlWKTm9pT82Q/videos

[link117] https://fuckscience.d3.ru/comments/676617/

[link118] https://ru.wikipedia.org/wiki/Гравитон

[link119] https://www.pgpru.com/comment80264

[link120] http://www.pgpru.com/comment56626

[link121] http://www.quantiki.org/wiki/Channel_(CP_map)

[link122] https://physics.stackexchange.com/questions/33078/what-is-a-completely-positive-map-physically

[link123] https://en.wikipedia.org/wiki/Quantum_operation

[link124] http://www.pgpru.com/biblioteka/statji/koblitcmatematikikriptografy

[link125] http://www.pgpru.com/comment36423

[link126] http://www.pgpru.com/comment48044

[link127] http://www.scottaaronson.com/blog/?p=2262

[link128] http://klnran.ru/2015/04/alexandrov1/

[link129] http://klnran.ru/2015/04/alexandrov2/

[link130] https://ru.wikipedia.org/wiki/Дальнодействующее_акустическое_устройство

[link131] http://www.rcrm.by/fizioterapiya.html

[link132] https://ru.wikipedia.org/wiki/Магнитными_материалами

[link133] https://ru.wikipedia.org/wiki/Агорафобия